17.09.2022 Views

Диссертация

You also want an ePaper? Increase the reach of your titles

YUMPU automatically turns print PDFs into web optimized ePapers that Google loves.

Ф.7.38-01

Министерство образования и науки Республики Казахстан

НАО «Южно-Казахстанский университет им. М. Ауэзова»

УДК 373.167.1:514

На правах рукописи

ЮЛДАШЕВА САОДАТ БЕКПУЛАТОВНА

Анализ простейших правил раскрытия неопределенностей и правила

Лопиталя

М010 Подготовка педагогов математики

7М01510 Математика научно-педагогическое

Диссертация на соискание степени магистра

Научный руководитель:

д.т.н., профессор

Адамов А. А.

Шымкент, 2022г.


СОДЕРЖАНИЕ

Введение............................................................................................. 3

1 ТЕОРЕТИЧЕСКИЕ ОСНОВЫ ТЕМЫ 6

1.1 Последовательность как функция натурального аргумента.

Предел числовой последовательности ………………………………… 6

1.2 Предел функции ………………………………………………. 12

2 ПРОСТЕЙШИЕ ПРАВИЛА РАСКРЫТИЯ

НЕОПРЕДЕЛЕННОСТЕЙ И ПРАВИЛА ЛОПИТАЛЯ 25

2.1 Программа элективного курса «Раскрытие

неопределенностей при вычислении пределов» ……………………….. 25

2.2 Содержание элективного курса «Раскрытие

неопределенностей при вычислении пределов» ……………………….. 29

2.3 Педагогический эксперимент .................................................... 70

ЗАКЛЮЧЕНИЕ ................................................................................. 74

СПИСОК ИСПОЛЬЗОВАННЫХ ИСТОЧНИКОВ ……………………. 75

ПРИЛОЖЕНИЕ А ....................................................................................... 77

2


ВЕДЕНИЕ

Актуальность исследования. Возросшая роль математики в жизни

современного общества поднимает значение математики как учебного

предмета в средней школе и выдвигает перед современной школы задачу

воспитания людей, способных не только извлекать из своей памяти готовые

знания, но и умеющих ориентироваться в нарастающем потоке научной

информации, владеющих общими идеями и методами, позволяющими

охватить с общей точки зрения многообразные факты и явления.

Анализ содержания школьного математического образования позволил

выявить ряд недостатков в обучении школьников:

1. Неподготовленность учащихся к пониманию современных научных

идей и их применению. Это объясняется тем, что учебный предмет алгебры в

настоящее время дает весьма узкое представление о теории пределов. Курс

алгебры средней школы содержит в основном материал, накопленный к

середине ХVII века, в котором переплетаются вопросы арифметики, алгебры

и анализа. Такое разнообразие вопросов курса не может создать у учащихся

представления о теории пределов.

2. Студенты, особенно на первых ступенях обучения, испытывают

серьезные трудности при обучении в вузе, поскольку познают большое число

новых понятий, в логическом отношении значительно более сложных, чем

школьные.

Перечисленные выше недостатки указывают на необходимость

внесения изменения в содержание школьного математического образования.

В своем исследовании мы предлагаем включить элементы математического

анализа, в том числе теории пределов в содержание школьного

математического обучения.

Основные понятия математического анализа, так или иначе,

складываются у учащихся, поскольку они в своих конкретных проявлениях

составляют основное содержание школьного куpca магематики. Поэтому

одной из задач на современном этапе является организация процесса

обучения основным понятиям теории пределов, что позволит:

- приблизить содержание школьного курса математики к современной

математике;

- повысить уровень математической подготовки учащихся;

- подготовить учащихся к продолжению образования в вузе;

- развить мышление учащихся.

Для того чтобы познакомить учащихся с элементами математического

анализа, в том числе теории пределов вовсе не обязательно вводить их в

основной курс математического анализа. В современных условиях работы

школ для этой цели наилучшим образом подходит элективный курс, который

является одним из средств повышения теоретической и практической

подготовки учащихся по математике.

3


Все вышеизложенное свидетельствует о необходимости разработки

содержания и методики изучения элементов математического анализа, в том

числе теории преелов на элективном курсе, а, следовательно, подтверждает

актуальность данного исследования.

Вопросы содержания элективного курса, основы его организации

является объектом исследования, начиная с момента их создания.

По нашему мнению, идея введения элементов теории пределов в

школьный курс математики состоит не столько в том, чтобы специально и

обособленно изучать вопросы теории пределов, а в том, что их изучение

позволит повысить уровень математического развития учащихся.

Проблема исследования заключается в поиске путей постановки

факультативного кypca. по изучению элементов теории пределов в старших

классах средней школы.

Цель исследования - методическое обоснование целесообразности

изучения элементов теории пределов на элективном курсе, разработка

содержания и методики изучения элективного курса «Раскрытие

неопределенностей при вычислении пределов».

Объект исследования: процесс обучения алгебры и начал анализа.

Предметом исследования является содержание учебного материала по

теме «Раскрытие неопределенностей при вычислении пределов» и методика

его изучения.

Гипотеза исследования: школьный курс математики располагает

большими возможностями для изучения элементов теории пределов в

старших классах. Элективный курс «Раскрытие неопределенностей при

вычислении пределов» будет способствовать повышению прочности знаний

по предмету, уровня интеллектуального развития школьников, подготовит

учащихся к пониманию современных научных идей и их применения, тем

самым приблизит школьное преподавание к современной науке и ее

приложениям.

Исходя из сформулированной гипотезы, для достижения цели

исследования необходимо было последовательно решить следующие задачи:

1. Провести анализ научной и учебно-методической литературы по

теме исследования.

2. Выявить связь понятий теории пределов с основным школьным

курсом математики.

3. Разработать структуру и содержание элективного курса «Раскрытие

неопределенностей при вычислении пределов» на уровне, адекватном для

учащихся старших классов.

4. Разработать методические рекомендации по изучению тем

элективного курса.

5. Провести экспериментальную проверку разработанных материалов.

Проблема, цели и задача исследования обусловили выбор следующих

методов исследования:

4


- изучение и анализ психолого-педагогической, научно-методической и

учебной литературы по теме исследования;

- изучение и обобщение опыта постановки элективных курсов;

- беседы и анкетирование учащихся, учителей, студентов;

- организация и проведение педагогического эксперимента;

- обработка данных, полученных в ходе эксперимента.

Научная новизна исследования состоит в разработке концепции,

содержания элективного курса «Раскрытие неопределенностей при

вычислении пределов» для учащихся старших классов и методических

рекомендаций по его изучению.

Теоретическая значимость исследования заключается в том, что

обоснована целесообразность и возможность изучения элементов теории

пределов в старших классах.

Практическая значимость исследования. Материалы исследования

могут быть использованы в практической деятельности учителей средних

школ, преподавателей вузов, а также студентами при написании курсовых и

дипломных работ, в процессе их педагогической практики. Результаты

работы найдут широкое распространение в системе повышения

квалификации учителей математики.

5


1 ТЕОРЕТИЧЕСКИЕ ОСНОВЫ ТЕМЫ

1.1 Последовательность как функция натурального аргумента. Предел

числовой последовательности

Числовая последовательность – функция вида

где N

множество натуральных чисел (или функция натурального аргумента),

обозначается

Значения

называют

соответственно первым, вторым, третьим, … членами последовательности.

Например, для функции можно записать:

y f n y1 y2

y n

( ) или , , , ,

y

n

2

6

y

f ( x), x N,

y , y , y ,

1 2 3

2 2 2 2

y1 1 1; y2 2 4; y3

3 9; yn

n ;

Способы задания последовательностей. Последовательности можно

задавать различными способами, среди которых особенно важны три:

аналитический, описательный и рекуррентный.

1. Последовательность задана аналитически, если задана формула ее n-

го члена:

y

n

f ( n).

y

2n1последовательность нечетных чисел: 1, 3, 5, 7, .…

Пример.

2. Описательный способ задания числовой последовательности состоит

в том, что объясняется, из каких элементов строится последовательность.

Пример 1. «Все члены последовательности равны 1». Это значит, речь

идет о стационарной последовательности 1, 1, 1, …, 1, ….

Пример 2. «Последовательность состоит из всех простых чисел в

порядке возрастания». Таким образом, задана последовательность 2, 3, 5, 7,

11, …. При таком способе задания последовательности в данном примере

трудно ответить, чему равен, скажем, 1000-й элемент последовательности.

3. Рекуррентный способ задания последовательности состоит в том, что

указывается правило, позволяющее вычислить n-й член последовательности,

если известны ее предыдущие члены. Название рекуррентный способ

происходит от латинского слова recurrere – возвращаться. Чаще всего в таких

случаях указывают формулу, позволяющую выразить n-й член

последовательности через предыдущие, и задают 1–2 начальных члена

последовательности.

Пример 1.

Здесь

Можно видеть, что полученную в этом примере последовательность

может быть задана и аналитически: yn

4n

1.

Пример 2.

Здесь: y1 1; y2 1; y3 11 2; y4 1 2 3; y5 2 3 5; y6

3 5 8;

Последовательность, составленную в этом примере, специально

изучают в математике, поскольку она обладает рядом интересных свойств и

приложений. Ее называют последовательностью Фибоначчи – по имени

итальянского математика 13 в. Задать последовательность Фибоначчи

n

y 3; y y 4, если n 2,3,4....

1 n n1

y 3; y 3 4 7; y 7 4 11;...

1 2 3

y 1; y 1; y y y , если n 3,4....

1 2 n n2 n1


рекуррентно очень легко, а аналитически – очень трудно. n-е число

Фибоначчи выражается через его порядковый номер следующей формулой

a n

n

1

1

5 1

5

5 2 2

n

.

На первый взгляд, формула для n-го числа Фибоначчи кажется

неправдоподобной, так как в формуле, задающей последовательность одних

только натуральных чисел, содержатся квадратные корни, но можно

проверить «вручную» справедливость этой формулы для нескольких первых

n.

Свойства числовых последовательностей.

Числовая последовательность – частный случай числовой функции,

поэтому ряд свойств функций рассматриваются и для последовательностей.

называют возрастающей, если

каждый ее член (кроме первого) больше предыдущего:

Определение. Последовательность

y1 y2 y3 ... yn

yn

1

....

Определение. Последовательность называют убывающей, если

каждый ее член (кроме первого) меньше предыдущего:

y1 y2 y3 ... yn

yn

1

....

Возрастающие и убывающие последовательности объединяют общим

термином – монотонные последовательности.

Пример 1.

возрастающая последовательность.

Пример 2.

Пример 3.

y y n

y

2

1

1; n

1

1

1; yn

убывающая последовательность.

n

y y n

1

n

1

1

1; ( 1) n

7

y n

y n

эта последовательность не является не

возрастающей не убывающей.

Определение. Последовательность называется периодической, если

существует такое натуральное число T, что начиная с некоторого n,

выполняется равенство y . Число T называется длиной периода.

n

yn T

Пример. Последовательность

периодична с длиной периода

T= 2.

Арифметическая прогрессия.

Числовую последовательность, каждый член которой, начиная со

второго, равен сумме предыдущего члена и одного и того же числа d,

называют арифметической прогрессией, а число d– разностью

арифметической прогрессии.

Таким образом, арифметическая прогрессия – это числовая

последовательность a n,

заданная рекуррентно соотношениями

a1 a, an

an

1 d ( n 2,3,4,...)

(a и d – заданные числа).

y ( 1) n

n


Пример. 1, 3, 5, 7, 9, 11, … – возрастающая арифметическая прогрессия,

у которой

Пример. 20, 17, 14, 11, 8, 5, 2, –1, –4,… – убывающая арифметическая

прогрессия, у которой

Нетрудно найти явное (формульное) выражение через n. Величина

очередного элемента возрастает на d по сравнению с предыдущим, таким

образом, величина n элемента возрастет на величину по сравнению с

первым членом арифметической прогрессии, т.е.

a1 1, d 2.

a a1 d( n 1).

n

a1 20, d 3.

8

a n

( n1)

d

Это формула n-го члена арифметической прогрессии.

Используя явное выражение через , можно доказать следующее

свойство арифметической прогрессии: если натуральные числа

таковы, что

Чтобы в этом убедиться,

a n

i j k l, то a a a a .

i, j, k и l

i j k l

n

i, j, k,

l

достаточно подставить

вместо в формулу

арифметической прогрессии и сложить. Отсюда следует, что если

рассматривать первые членов арифметической прогрессии, то суммы

членов, равно отстоящих от концов, будут одинаковы:

1 n 2 n1 3 n2 1

n

a a a a a a ... 2 a ( n 1) d.

n

n -го члена

Последнее равенство позволяет вычислить сумму первых n членов

арифметической прогрессии:

S a a ... a a .

n 1 2 n1

n

С этой целью берется еще одна такая же сумма, но слагаемые

записывается в обратном порядке:

S a a ... a a .

n n n1 2 1

Далее она складывается почленно с исходной суммой, причем

слагаемые сразу попарно группируются. В результате

2 S ( a a ) ( a a ) ... ( a a ) n(2 a ( n 1) d),

n 1 n 2 n1 n 1 1

откуда

S

n

n a

2

(

1

an) .

Это формула суммы

n

членов арифметической

прогрессии.

Арифметической прогрессия названа потому, что в ней каждый член,

кроме первого, равен среднему арифметическому двух соседних с ним –

предыдущего и последующего. Действительно, так как

a a d; a a d.

n n1 n n1

Сложение двух последних равенств дает

a

a

2

n1 n

a 1

n

.

Таким образом, верна следующая теорема (характеристическое

свойство арифметической прогрессии). Числовая последовательность

является арифметической тогда и только тогда, когда каждый ее член, кроме

первого (и последнего в случае конечной последовательности), равен

среднему арифметическому предшествующего и последующего членов.


Пример. При каком значении числа 3 2,5 4 и 11 12 образуют

конечную арифметическую прогрессию?

Согласно характеристическому свойству, заданные выражения должны

удовлетворять соотношению

5x 4 ((3x 2) (11x

12)) / 2.

x

9

x x x

Решение этого уравнения дает

заданные

выражения

принимают, соответственно, значения

Это – арифметическая прогрессия, ее разность равна

14,5, 31,5, 48,5.

17.

3x 2,5x 4 и 11x

12

x 5,5. При этом значении

Геометрическая прогрессия.

Числовую последовательность, все члены которой отличны от нуля и

каждый член которой, начиная со второго, получается из предыдущего члена

умножением на одно и то же число , называют геометрической

прогрессией, а число q – знаменателем геометрической прогрессии.

Таким образом, геометрическая прогрессия – это числовая

последовательность , заданная рекуррентно соотношениями

1 n n1

b n

b b, b b q( n 2,3,4...).

b и q

b0, q0

( – заданные числа,

).

Пример 1. 2, 6, 18, 54, … – возрастающая геометрическая

прогрессия b = 2, q = 3.

Пример 2. 2, –2, 2, –2, …–геометрическая прогрессия b= 2, q= –1.

Пример 3. 8, 8, 8, 8, …–геометрическая прогрессия b= 8, q= 1.

Геометрическая прогрессия является возрастающей

b1 0, q1,

q

x

b1 0,0 q1.

последовательностью, если

и убывающей, если

Одно из очевидных свойств геометрической прогрессии состоит в том,

что если последовательность является геометрической прогрессией, то и

последовательность квадратов, т.е.

2 2 2

b1 b2 b3

b n

, , ,..., 2,...

является геометрической прогрессией, первый член

которой равен

Формула n -го члена геометрической прогрессии имеет вид

b

n

b q

n 1

1

.

b 2

, 1

а знаменатель

q 2 .

Можно получить формулу суммы членов конечной геометрической

прогрессии.

Пусть дана конечная геометрическая прогрессия

b1 b2 b3

b n

, , ,..., .

S

Пусть сумма ее членов, т.е.

Sn

b1 b2 b3 ... bn.

Принимается, что q Для определения S

n применяется искусственный

прием: выполняются некоторые геометрические преобразования выражения

Sq

n

.

Тогда

n


S q ( b b b ... b b ) q b b b ... b b q S b q b .

n 1 2 3 n1 n 2 3 4 n n n n 1

Таким образом,

S

n

n

b1 q 1 .

q 1

q 1.

S

n

S q S b q b

n n n

1

и, следовательно,

Это формула суммы n членов геометрической прогрессии для случая,

когда

При

случае

Геометрической прогрессия названа потому, что в ней каждый член

кроме первого, равен среднему геометрическому предыдущего и

последующего членов. Действительно, так как

q 1 формулу можно не выводить отдельно, очевидно, что в этом

a n

1 .

b b q; b b / q,

n n1 n n1

b 2 b b

следовательно,

и верна следующая теорема

(характеристическое свойство геометрической прогрессии):

числовая последовательность является геометрической прогрессией

тогда и только тогда, когда квадрат каждого ее члена, кроме первого (и

последнего в случае конечной последовательности), равен произведению

предыдущего и последующего членов.

n n1 n1

Предел последовательности.

Пусть есть последовательность

c

n

Эту последовательность

называют гармонической, поскольку каждый ее член, начиная со второго,

есть среднее гармоническое между предыдущим и последующим членами.

Среднее геометрическое чисел

n

a и b есть число

n

1/ .

2ab

c ,

a b

c 2 a b

или

1 1 1 1 .

С ростом все члены геометрической прогрессии убывают и их значение

приближается к нулю. В этом случае принято говорить, что при n ,

стремящемся к бесконечности, данная последовательность сходится и нуль

есть ее предел. Записывается это так:

1

lim 0.

n

n

Строгое определение предела формулируется следующим образом:

Если существует такое число A , что для любого (сколь угодно малого)

положительного числа найдется такое натуральное (вообще говоря,

зависящее от ), что для всех n N будет выполнено неравенство

an

A ,

то говорят, что последовательность a

n сходится и

предел.

Обозначается это так: lim.

n

a

n

A

В противном случае последовательность называется расходящейся.

N

A – ее

10


Опираясь на это определение, можно, например, доказать наличие

предела

Пусть –

сколь угодно малое положительное число. Рассматривается разность

1 1

an

A 0 .

n n

Существует ли такое N , что для всех n N выполняется неравенство

. Если взять в качестве N любое натуральное число, превышающее

то для всех выполняется неравенство

что и

требовалось доказать.

Доказать наличие предела у той или иной последовательности иногда

бывает очень сложно. Наиболее часто встречающиеся последовательности

хорошо изучены и приводятся в справочниках. Имеются важные теоремы,

позволяющие сделать вывод о наличии предела у данной последовательности

(и даже вычислить его), опираясь на уже изученные последовательности.

Теорема 1. Если последовательность имеет предел, то она ограничена.

Теорема 2. Если последовательность монотонна и ограничена, то она

1/ N

1/ ,

A 0 у гармонической последовательности

n

N

имеет предел.

Теорема 3. Если последовательность

последовательности

can , an c и an

соответственно (здесь c– произвольное число).

Теорема 4. Если последовательности

a n

имеют

равные Aи

B соответственно, то последовательность

предел pA qB.

Теорема 5. Если последовательности и

равные Aи

B соответственно, то последовательность

AB .

Теорема 6. Если последовательности и

равные

B

последовательность

соответственно, и, кроме того,

имеет предел [1, 2].

a

n

/ b

n

a n

a n

a n

A/ B.

и

c

n

1/ n .

1/ n/ N,

имеет предел

пределы

b n

b n

b n

ab

n

b

n

cA, A+c,

A, то

A

имеют пределы,

pa

n

qb

n

имеет

имеют пределы,

n

имеет предел

имеют пределы,

0 и B0,

то

11


1.2 Предел функции

Понятие функции. Рассматривая реальные физические переменные

величины, мы приходим к выводу, что эти величины всегда могут принимать

произневольные значения. Так, температура тела не

скорость

материальной точки не может быть меньше может быть больше

(т. е. скорости света в пустоте), смещение у материальной

точки, совершающей гармонические колебания по закону

может изменяться лишь в пределах сегмента A, A.

B математике отвлекаются от конкретных физических свойств

наблюдаемых в природе переменных величин и рассматривают абстрактную

переменную величину ( уместно отметить, что понятие величины относится к

числу начальных математических понятий) , характеризуемую только

численными значениями, которые она может принимать.

Множество {x} всех значений, которые может принимать данная

переменная величина, называется областью изменения этой переменной

величины. Переменная величина считается заданной, если задана область ее

изменения. В дальнейшем мы, как правило, будем обозначать переменные

величины маленькими латинскими буквами а области изменения

10

310 см/сек

этих переменных символами

x , y , u ,...

Пусть задана переменная величина

некоторое множество .

x

x

x, y, u,...,

Если каждому значению переменной

соответствие по известному закону некоторое число

множестве задана

x

y y( x) или y f ( x).

273 С,

y Asin( t

),

, имеющая областью изменения

x

из множества x

ставимся в

y,

то говорят, что на

При этом переменная x называется аргументом, а множество -

областью задания функции

Число , которое соответствует данному значению аргумента

называется частным значением функции в точке

частных значений функции образует вполне определенное множество y

,

называемое множеством всех значений функции.

В обозначении

буква называется характеристикой

функции. Для обозначения аргумента, функции и ее характеристики могут

употребляться различные буквы. Приведем примеры функций:

1°. Эта функция задана на бесконечной прямой

Множество всех значений этой функций - полупрямая 0 .

( рис.1 )

y

y

x 2 .

y

y

f ( x)

f ( x).

f

x

x. Совокупность всех

y

x

x .

12


y

0

Рисунок 1- График функции

x

2

2. y 1 x . Функция задана на сегменте

значений этой функции-сегмент 0 1. ( рис.2 )

y

1

x 1.

Множество всех

y

-1 0

1

x

Рисунок 2- График функции

3. Эта функция задана на множестве натуральных чисел

n 1,2,...

Множество всех значений этой функции - множество натуральных

чисел вида

y

y

n!.

n!. ( рис.3 ).

4!

3!

2!

1!

x

0 1 2 3 4

Рисунок 3- График функции y

n!.

13


4. Функция Дирихле

0, если х иррациональное число,

y

1, если х рациональное число.

Эта функция задана на бесконечной прямой

всех ее значений состоит из двух точек 0 и 1.

5°.

1, если x>0,

ysgn x 0, если x=0,

1, если x<0.

x ,

а множество

(Термин sgn происходит от латинского слова signum - знак) Эта

функция задана на всей бесконечной прямой x , а множество всех

ее значений состоит их трех точек: 1,0 и 1. ( рис.4 ).

y

1

0

x

y

[ x]

-1

Рисунок 4- График функции

y

sgn x

6. где [] x обозначает целую часть вещественного числа x .

Читается: " y равно антье

функция задана для всех вещественных значений х , а множество всех ее

значений состоит из целых чисел. ( рис.5 ).

y

х" (от французского слова entier - целый). Эта

-

-

-

-

4

3

2

1

Рисунок 5- График функции y

[ x]

-

-

-

-

1 2 3 x 4

5

x

14


2. О способах задания функции. В этом пункте мы остановимся на

некоторых способах задания функции.

Часто закон, устаналивающий связь между аргументом и функцией,

задается посредством формул. Такой способ задания функции называется

аналитическим. Следует подчеркнуть, что функция может определяться

разными формулами на разных участках области своего задания.

Например, функция

sin x при x

0,

y 2

x

при x

0

задана аналитическим способом на всей бесконечной прямой.(рис.6)

y

y = x 2

y = sinx

0

x

sin x при x

0,

Рисунок 6-График функции y 2

x

при x

0

Довольно распространенным способом задания функции является

табличный способ, заключающийся в задании таблицы отдельных значений

аргумента и соответствующих им значений функции. При этом можно

приближенно вычислить не содержащиеся в таблице значения функции,

соответствующие промежуточным значениям аргумента. Для этого

используется способ интерполяции, заключающийся в замене функции

между ее табличными значениями какой-либо простой функцией (например,

линейной или квадратичной). Примером табличного задания функции может

служить расписание движения поезда. Расписание определяет

местоположение поезда в отдельные моменты времени. Интерполяция

позволяет приближенно определить местополо жение поезда в любой

промежуточный момент времени.

В практике физических измерений используется и еще один способ

задания функции - графический, при котором соответствие между

аргументом и функцией задается посредством графика (снимаемого,

например, на осциллографе).

Понятие предельного значения функции

1. Определение предельного значения функции. Рассмотрим функцию

y f ( x),

определенную на некотором множестве x,

и точку a , быть

может, и не принадлежащую множеству x,

но обладающую тем свойством,

15


что в любой окрестности точки a имеются точки множества

x,

отличные от

может быть граничной точкой интервала,

на котором определена функция.

Определение 1. Число

y

f ( x)

сходящейся к

a. Например, точка a

b называется предельным значением функции

в точке x a (или пределом функции при

x n

a

последовательности

16

x1 x2

x n

, ,... ,...

а

x

a

,

x

n

), если для любой

значений аргумента

элементы которой отличны oт

соответствующая

последовательность

значений функции сходится к b .

Для обозначения предельного значения функции используется

следующая символика:

f x1 f x2

f x n

( ), ( ),... ( ),...

lim f ( x ) b .

x a

Отметим, что функция

y

f ( x)

a

может иметь в точке

предельное значение. Это вытекает из того, что последовательность

может иметь только один предел. Рассмотрим несколько примеров.

1. Функция

f ( x)

c

a

х,

только одно

f x n

имеет предельное значение в каждой точке

x , x ,... x ,...

бесконечной прямой. В самом деле, если

любая сходящаяся к

последовательность значений аргумента, то соответствующая

последовательность значений функции имеет вид

..... и поэтому

сходится к

точке x a равно c .

a

1 2

n

c, c,..., c,...

c. Таким образом, предельное значение этой функции в любой

f ( x)

2. Предельное значение функции

в любой точке

бесконечной прямой равно . Действительно, в этом случае

последовательности значений аргумента и функции тождественны, и

поэтому, если последовательность

f x n

последовательность

a

x n

также сходится к a .

x

сходится

к

a

a

a

, то и

3. Функция Дирихле, значение которой в рациональных точках равны

единице, а в иррациональных - нулю, не имеет предельного значения ни в

одной точке a бесконечной прямой. Действительно, для сходящейся к a

последовательности рациональных значений аргумента предел

соответствующей последовательности значений функции равен единице, а

a

для сходящейся к последовательности иррациональных значений

аргумента предел соответствующей последовательности значений функции

равен нулю.

В дальнейшем мы будем использовать понятия односторонних

предельных значений функции, которые определяются следующим образом.

Определение 2. Число b называется правым (левым) предельным

значением функции

f( x)

в точке x a,

если для любой сходящейся к a

последовательности

x1, x2,... x n

,...

значений аргумента x , элементы

xn


a

которой больше (меньше) , соответствующая последовательность

значений функции сходится к b .

Для правого предельного значения функции используется обозначение

f x1 f x2

f x n

( ), ( ),... ( ),...

lim f ( x) b или f ( a 0) b.

xa0

Для левого предельного значения употребляется обозначение

lim f ( x) b или f ( a 0) b.

xa0

В качестве примера рассмотрим функцию

Эта функция

имеет в нуле правое и левое предельные значения, причем

sgn(0 0) 1, а sgn(0 0) 1.

В самом деле, если

17

f ( x) sgn x.

x n

- любая сходящаяся к

нулю последовательность значений аргумента этой функции, элементы

которой больше нуля x 0, n

то sgn x 1 n

limsgn xn

1.

и поэтому n

Таким

образом, справедливость равенства

доказывается, что

sgn(0 0) 1.

Замечание. Если в точке

f( x)

a

a

sgn(0 0) 1

x n

установлена. Аналогично

правое и левое предельные значения

функцuu равны, тo в точке существует предельное значение этой

функции, равное указанным односторонним предельным значениям. Этот

наглядный факт мы снабдим доказательством.

Пусть

x n

- любая сходящаяся к

a

последовательность значений

аргумента функции

f( x),

элементы которой не равны . Пусть

подпоследовательность этой последовательности, состоящая из всех

элементов последовательности

x n

, a

x lm

a

x

k m

- подпоследовательность,

состоящая из всех меньших a элементов последовательности

исключаем из рассмотрения случай, когда у последовательности

x n

x n

(Мы

лишь

конечное число элементов лежит правее (левее) точки a . В этом случае

сходимость

и

x lm

{ f( x n

)}

сходятся к

значений функции

f x lm

и

очевидна). Так как в силу подпоследовательности

a

, то из существования правого и левого предельных

f( x)

в точке

a

x km

вытекает, что последовательности

f x km

имеют пределы, которые по условию равны. Пусть

предел этих последовательностей. Для любого 0 можно указать номер N

такой, что все элементы последовательностей

которых

km

N и lm

N,

удовлетворяют

k l

f x b и f x b .

m

m

f x km

и

f x lm

b

, для

неравенствам

Следовательно, при n N выполняется


неравенство

f ( xn)

b

т. е. последовательность

( )

f x n

сходится к b . Тем

f( x)

самым доказано, что предельное значение функции в точке а

существует и равно b.

Сформулируем определения предельного значения функции при

стремлении аргумента к бесконечности и к бесконечности определенного

знака.

Определение 3. Число

f( x)

x

b называется предельным значением функции

x (или пределом функции при

x ), если для любой

при

бесконечно большой последовательности значений аргумента

соответствующая последовательность значений функции сходится к b .

Для обозначения предельного значения функции при

используется следующая символика:

f( x)

lim f ( x) b.

x

x

Определение 4. Число b называется предельным значением функции

x

при стремлении аргумента к положительной (отрицательной)

бесконечности, если для любой бесконечно большой последовательности

значений аргумента, элементы которой, начиная с некоторого номера,

положительны (отрицательны), соответствующая последовательность

значений функции сходится к b.

Символические обозначения:

lim f ( x) b ( lim f ( x) b).

x

x

В качестве примера рассмотрим функцию

равное нулю предельное значение при

x , x ,... x ,...

1 2

n

1

f( x) .

x

Эта функция имеет

х . Действительно, если

бесконечно большая последовательность значений аргумента,

1 1 1

, ,... ,...

x1 x2

x n

то последовательность

бесконечно малая и поэтому имеет

предел, равный нулю.

2. Арифметические операции над функциями, имеющими предельное

значение. Убедимся, что арифметические операции над функциями,

имеющими предельное значение в точке а , приводит функциям, также

имеющим предельное значение в этой точке. Справедлива следующая

основная теорема.

Теорема 1. Пусть заданные на одном и том же множестве функции

f ( x) и g( x)

имеют в точке предельные значения b и c . Тогда функции

f( x)

f ( x) g( x), f ( x) g( x), f ( x) g( x) и

gx ( ) имеют в точке a предельные

18


значения (частное при условии

b

b c, b c, b c

и .

c

а

Доказательство. Пусть

x , 1

x ,... ,...

2

xn

xn

а

19

c 0 ), равные соответственно

произвольная сходящаяся к

f ( x) и g( x).

последовательность значений аргумента функций

Соответствующие

последовательности

f ( x ), f ( x ),... f ( x ),... и g( x ), g( x ),... g( x ),...

1 2 n

1 2

пределы

b и c.

значений этих функций имеют

Но тогда, в силу теорем, последовательности

f x g x , f x g x , f x g x и

n n n n n n

соответственно

последовательности

равные

x n

b

b c, b c, b c

и .

c

это

n

означает,

lim f ( x) g( x) b c, lim f ( x) g( x) b c,

xa xa

f x

g x

n

n

имеют пределы,

В силу произвольности

lim f ( x) g( x) b c,

xa

f ( x)

b

lim .

xa

g( x)

c

Теорема доказана.

Применим доказанную теорему для отыскания предельных значений

многочленов и несократимых алгебраических дробей (Несократимая

алгебраическая дробь - частное двух многочленов, не имеющих отличных от

постоянной общих множителей). Имеет место следующее утверждение.

а

В каждой точке бесконечной прямой предельные значения

многочленов и несократимых алгебраических дробей существуют и равны

частным значениям этих функций в указанной точке (в случае

алгебраической дроби а не должно быть корнем знаменателя).

Действительно, в силу теоремы 4.1

2 2

lim x lim x x lim x lim x a .

xa xa xa xa

Аналогично можно убедиться, что

lim x n n

a .

x a

b x b x ... b x b

n n 1

0 1 n1

Следовательно для многочлена

получим

(используя теорему 1 для произведения и суммы)

1 1

lim b x n b x n

0 1

... b x b n n

n 1 n b a b a

0 1

... b a b

n1

n.

xа В случае несократимой алгебраической дроби, когда а не является

корнем знаменателя, получим (применяя теорему 4.1 для частного)

n n1 n n1

b0 x b1 x ... bn 1x bn b0a b1a ... bn 1a bn

lim

.

m m1 m m1

c0x c1x ... cm

1x cm c0a c1a ... cm

1a cm

3. Сравнение бесконечно малых и бесконечно больших функций.

Функция y f ( x)

называется бесконечно малой в точке xa ( при x a),

n


если lim f ( x

m

) 0. Легко убедиться, например, что функция f ( x) ( x a) ,

xa где

В самом деле, в предыдущем пункте мы установили, что предельное

значение многочлена

в любой точке бесконечной прямой

существует и равно частному значению многочлена в этой точке. Поэтому

x

a.

m целое положительное число, является бесконечно малой в точке

lim( x m

a ) 0.

x a

f ( x) ( x a) m

y

f ( x)

Отметим, что если функция

имеем равное предельное

значение в точке a , то функция

является бесконечно малой в

точке a . Действительно, предельные значения каждой из функций и

равны b, и поэтому в силу теоремы 1

[3, 4].

b в точке a

a( x) f ( x)

b

lim ( x) lim( f ( x) b) lim f ( x) limb

0.

xa xa xa xa

b

y

f ( x)

Используя полученный результат, мы получаем специальное

представление для функции, имеющей равное b предельное значение в точке

x

a:

f ( x) b ( x), где lim ( x) 0

Представление

xa

f ( x) b ( x), где lim ( x) 0

xa

оказывается

весьма

удобным при доказательстве различных предложений и будет неоднократно

использовано нами ниже.

Наряду с понятием бесконечно малой функции часто используется

понятие функции, бесконечно большой в точке справа или бесконечно

большой в точке а слева. Именно, функция называется бесконечно

большой в точке справа (слева), если для любой сходящейся к

последовательности

значений аргумента x , элементы

которой больше (меньше

f ( x1), f ( x2),..., f ( x

n)...

значений

функции является бесконечно большой последовательностью

определенного знака.

Для бесконечно больших функций используются следующие

обозначения:

xa0

a

а

x1 x2

x n

, ,..., ...

lim f ( x) или f ( a 0) ,

lim f ( x) или f ( a 0) ,

xa0

lim f ( x) или f ( a 0) ,

xa0

lim f ( x) или f ( a 0) .

xa0

y

а

f ( x)

a ), cоответствующая последовательность

Познакомимся с методикой сравнения бесконечно малых функций и

употребляемой терминологией.

Пусть ( x) и ( x)

две заданные на одном и том же множестве функции,

являющиеся бесконечно малыми в точке x

a.

а

x n

20


( x)

1) функция называется бесконечно малой более высокого

порядка, чем ( x)

(имеет более высокий порядок малости), если предельное

значение функции

( x)

( x)

в точке a равно нулю.

( x) и ( x)

2) функции

называются бесконечно малыми одного

порядка (имеют одинаковый порядок малости), если предельное значений

функции

( x)

( x)

в точке а

существует и отлично от нуля.

( x) и ( x)

3) функции

называются эквивалентными бесконечно

( x)

малыми, если предельное значение функции в точке а равно единице.

( x)

Часто бесконечно малые функции сравнивают с какими-либо

стандартными бесконечно малыми функциями. Обычно в качестве функции

сравнения берут функцию где m целое положительное число. В

этом случае употребляется следующая терминология: бесконечно малая в

точке a функция имеет порядок малости m , если предельное значение

функции

( x)

( x

a) m

( x)

в точке a

m

( x

a) ,

отлично от нуля.

При сравнении бесконечно малых функций часто употребляют символ

малое). Именно, если функция представляет собой бесконечно

малую в точке a функцию более высокого порядка, чем бесконечно малая в

этой же точке функция то это условно записывают так:

o

(o

0

( x),

( x)

(читается: равно малое от ). Таким образом, символ o( )

означает любую бесконечно малую функцию, имеющую в точке a более

высокий порядок малости, чем бесконечно малая в этой точке функция

( x).

a

o

Отметим следующие очевидные свойства символа о : если

o( ) o( ) o .

то o( ) o( ) o

,

o( ),

Заметим также, что если

функции,

то функция имеет более высокий порядок малости, чем каждый из

сомножителей, и поэтому

o( ), o( ).

и бесконечно малые в точке a

Для бесконечно больших в точке

используется аналогичная методика сравнения.

Пусть

- бесконечно большие в точке а справа функции, и

пусть, например, обе эти бесконечно большие функции положительного

знака, т.е.

A( x) и B( x)

a справа (или слева) функций

21


lim A( x) и lim B( x) .

xa0 xa0

Ax ( )

Мы будем говорить, что функция имеет в точке справа более

высокий порядок роста, чем функция Bx ( ), если функция является

бесконечно большой в точке справа. Если же правое предельное значение

функции в точке конечно и отлично от нуля, то в этом случае мы будем

говорить, что

имеют в точке справа одинаковый порядок

роста.

Рассмотрим несколько примеров.

2 3 2

1. Функции ( x) 3 x x и ( x) 2x

являются бесконечно малыми

функциями одного порядка в точке

( x) 3 1

( x) 3

x 0 x.

Так как

то в силу теоремы 4.1 lim .

( x) 2 2

x0

( x) 2

А это означает, что

- бесконечно малые одного порядка.

2. Функции

малые в точке

теоремы 4.1

( x) и ( x).

точке

3. Функции

a

A( x) и B( x)

a

1

lim x 0,

x0

2

( x) и ( x)

( x) x 6 x и ( x) 2x

x 0. В самом деле,

( x)

lim 1.

x0

( x)

а

a

x 0. Действительно, при

2 3 2

эквивалентные бесконечно

( x)

16 x.

( x)

Так как

lim6x

0, то в силу

x0

Это и означает эквивалентность бесконечно малых

1

x 1

A( x) и B( x)

x x

x 0 справа и слева. Это следует из того, что

имеют одинаковый порядок роста в

Ax ( )

lim lim(1 x) 1.

x0 Bx ( ) x0

Понятие непрерывности функции.

1. Определение непрерывности функции. Пусть точка принадлежит

области задания функции и любая окрестность точки содержит

отличные от а точки области задания этой функции.

Определение 1. Функция f( x ) называется непрерывной в точке а , если

предельное значение этой функции в точке существует и равно частному

значению

Таким образом, условие непрерывности функции в точке а

символически можно выразить следующим образом:

lim f ( x ) f ( a ).

xa Так как a lim x,

то этому равенству можно придать следующую

f ( а).

xa

форму:

lim f ( x) f (lim).

xa xa

f( x)

а

а

f( x)

Следовательно, для непрерывной функции символ "lim" предельного

перехода и символ " f " характеристики функции можно менять местами.

а

22


f( x)

Определение 2. Функция называется непрерывной справа (слева)

в точке а , если правое (левое) предельное значение этой функции в точке

существует и равно частному значению

Символические обозначения непрерывности справа (слева):

lim f ( x) f ( a) или f ( a 0) f ( a)

xa0

lim f ( x) f ( a) или f ( a 0) f ( a)

xa0

f( x)

f ( а).

Замечание. Если функция непрерывна в точке a и слева и справа,

то она непрерывна в этой точке. В самом деле, в силу замечания п. 1 § 2 этой

главы в этом случае существует предельное значение функции в точке а,

равное частному значению этой функции в точке а [5, 6].

Рассмотрим примеры.

1°. Степенная функция

с целочисленным положительным

показателем и непрерывна в каждой точке бесконечной прямой.

Действительно, мы доказали, что предельное значение этой функции в любой

точке бесконечной прямой равно частному значению

2°. Так как многочлены и несократимые алгебраические дроби имеют в

каждой точке области задания предельное значение, равное частному

значению , то они являются непрерывными функциями.

Точки, в которых функция не обладает свойством непрерывности,

называются точками разрыва функции . Например, функция

имеет разрыв в точке

каждой точке бесконечной прямой, поскольку она не имеет предельного

значения ни в одной точке этой прямой .

Мы будем говорить, что функция непрерывна на множестве {x},

если она непрерывна в каждой точке этого множества. Если функция

непрерывна в каждой точке интервала, то говорят, что она непрерывна на

интервале. Если функция непрерывна в каждой внутренней точке сегмента

[a, b] и, кроме того, непрерывна справа в точке и слева в точке , то

говорят, что она непрерывна на сегменте [a, b].

2. Арифметические операции над непрерывными функциями.

Убедимся, что арифметические операции над непрерывными функциями

приводят к непрерывным функциям.

Докажем следующую основную теорему.

Теорема 2. Пусть заданные на одном и том же множестве функции

непрерывны в точке . Тогда функции

f ( x) sgn x

f ( x) и g( x)

f ( x) g( x), f ( x) g( x), f ( x) g( x),

ga ( ) 0).

f ( x)

x

n

f( x)

gx ( )

a

n

.

x 0 Функция Дирихле разрывна в

f( x)

а

a

непрерывны в точке a

b

а

(частное при

условии

Доказательство. Так как непрерывные в точке a функции f ( x) и g( x )

23


имеют в этой точке предельные значения f (a) и g (a). то в силу теоремы 1

предельные значения функций

f ( x) g( x), f ( x) g( x), f ( x) g( x) и

f( x)

gx ( )

существуют и равны соответственно

f ( а)

f ( а) g( а), f ( а) g( а), f ( а) g( а), .

g( а)

частным значениям перечисленных функций в точке

Hо эти величины как раз и равны

а. Теорема доказана [7].

24


2 ПРОСТЕЙШИЕ ПРАВИЛА РАСКРЫТИЯ

НЕОПРЕДЕЛЕННОСТЕЙ И ПРАВИЛА ЛОПИТАЛЯ

2.1 Программа элективного курса «Раскрытие неопределенностей

при вычислении пределов»

В этом разделе приведем программу элективного курса «Раскрытие

неопределенностей при вычислении пределов»

Аннотация

Актуальность темы. Значительное место в школьном курсе математики

занимают элементы математического анализа, в том числе и пределы

функций с раскрытием неопределенностей. Понятие предела функции

являются фундаментальным математическим понятием, используемым в

различных областях современной науки и техники. Целью изучения в

школьной программе этой темы является формирование интеллектуального

развития учащихся, формирование качеств мышления, необходимых

человеку для свободной ориентации в современном мире; овладение

математическими знаниями, необходимыми для применения в практической

деятельности, для изучения смежных дисциплин, для продолжения

образования. Но как показывает опыт преподавания учителей в школе,

вычисление пределов вызывает большие затруднения у школьников по

сравнению с другими темами. В разделе «Предел функции и непрерывность»

заметен высокий уровень научности и строгости понятий предела и

непрерывности функции. В основном материалы представлены для изучения

в высших учебных заведениях. В курсе же 10 класса отводится всего лишь 10

часов на раздел. Вследствие этого при обучении школьников данного раздела

учащиеся и учителя нуждаются в методических рекомендациях.

Предлагаемый элективный курс адресован учащимся 10 и 11 классов.

Главная его идея – это организация систематического и системного

повторения, углубления и расширения школьного курса математики, что,

несомненно, будет направлено на осмысленное изучение математики.

Данный курс позволит удовлетворить образовательные потребности

учащихся.

Пояснительная записка

Программа данного элективного курса ориентирована на рассмотрение

отдельных вопросов математики, которые входят в содержание образования.

Курс дополняет и развивает школьный курс математики, а также является

информационной поддержкой дальнейшего образования и ориентирован на

удовлетворение образовательных потребностей старших школьников, их

аналитических и синтетических способностей. Основная идея данного

элективного курса заключена в расширении и углублении знаний учащихся

по некоторым разделам математики, в обеспечении прочного и сознательного

овладения учащимися системой математических знаний и умений,

необходимых при сдаче выпускного экзамена, а для некоторых школьников -

необходимых для продолжения образования.

25


В процессе освоения содержания данного курса ученики овладевают

новыми знаниями, обогащают свой жизненный опыт, получают возможность

практического применения своих интеллектуальных, организаторских

способностей, развивают свои коммуникативные способности, овладевают

общеучебными умениями. Освоение предметного содержания курса и сам

процесс изучения его становятся средствами, которые обеспечивают переход

от обучения учащихся к их самообразованию.

Изучение курса предполагает обеспечение положительной мотивации

учащихся на повторение ранее изученного материала, выделение узловых

вопросов курса, предназначенных для повторения, использование схем,

моделей, опорных конспектов, справочников, тестов. Методологической

основой предлагаемого курса является деятельностный подход к обучению

математике. Данный подход предполагает обучение не только готовым

знаниям, но и деятельности по приобретению этих знаний, способов

рассуждений, доказательств. В связи с этим в процессе изучения курса

учащимся предлагаются задания, стимулирующие самостоятельное открытие

ими математических фактов, новых, ранее неизвестных, приемов и способов

решения задач.

Цель данного курса: обеспечение индивидуального и

систематического сопровождения учащихся при обучении разделу «Предел

функции и непрерывность» по алгебре и началам анализа.

Задачи курса:

1.Расширение и углубление школьного курса математики.

2.Актуализация, систематизация и обобщение знаний учащихся по

математике.

3.Формирование у учащихся понимания роли математических знаний как

инструмента, позволяющего выбрать лучший вариант действий из многих

возможных.

4.Развитие интереса учащихся к изучению математики.

5.Расширение научного кругозора учащихся.

6.Обучение старшеклассников решению учебных и жизненных проблем,

способам анализа информации, получаемой в разных формах.

Форма обучения:

Индивидуальная, парная, фронтальная, групповая.

Организация на занятиях элективного курса должна существенно

отличаться от урочной: учащемуся необходимо давать достаточное время на

размышление, приветствовать любые попытки самостоятельных

рассуждений, выдвижения гипотез, способов решения задач. В курсе

заложена возможность дифференцированного обучения.

Методы обучения:

Репродуктивные, поисковый, исследовательский.

Применяются следующие виды деятельности на занятиях: обсуждение,

тестирование, исследовательская деятельность, диспут, обзорные лекции,

семинары и практикумы по решению задач, предусмотрены консультации.

26


Формы контроля:

Проверочные работы, контрольная работа.

Для текущего контроля на занятиях учащимся рекомендуется серия

заданий, часть которых выполняется в классе, а часть - дома самостоятельно.

Количество заданий в тестах по каждой теме не одинаково, они носят

комплексный характер, и большая часть их призвана выявить уровень

развития математического мышления тестируемого. Основным

дидактическим средством для предлагаемого курса являются тексты

рассматриваемых типов задач, которые могут быть выбраны из

разнообразных сборников, различных учебных пособий или составлены

самим учителем.

Тематический план курса

п/п

Тема занятия

Литература

1. Абылкасымова А.Е., Кучер Т.П., Корчевский В.Е., Жумагулова З.А.,

Алгебра и начала анализа: Учебник для 10 класса ЕМН, Алматы: Мектеп,

2019;

2. Темиргалиев Н., Введение в математический анализ, Астана, 2015;

27

Кол-во

часов

1 Введение 1

2 Последовательность как функция натурального аргумента. 1

Предел числовой последовательности

3 Предел функции 2

4 Бесконечно малые и бесконечно большие функции 2

5 Непосредственное применение теорем о пределах.

3

Раскрытие неопределенностей.

6 Два замечательных предела. 3

7 Некоторые следствия замечательных пределов. 2

8 Раскрытие неопределенности по правилу Лопиталя

8.1. Расскрытие неопределенности вида

8.2. Неопределенности вида

8.3. Неопределенности других видов:

0 0

(0 ),( ),(1 ),(0 ),( ).

9 Разные задачи на вычисление пределов 10

10 Контрольная работа 1

Всего часов 34

0

0

3

3

3


3. Круглов Е.В., Мамаева Н.А., Таланова Е.А., Некоторые приемы

вычисления пределов Нижний Новгород, 2018;

4. Матвеева Т.А., Рыжкова Н.Г., Математический анализ, Екатеринбург,

2017;

5. Самочернова Л.И., Высшая математика, Томск, 2005;

6. Альпин Т.Ю., Егоров А.И., Кашаргин П.Е., Сушков С.В., Практические

занятия по математическому анализу, Казань, 2013.

7. Берман Г.Н. Сборник задач по курсу математического анализа. Издание

22-е, переработанное.- СПб.: Изд-во «Профессия», 2001. – 432с.

8. Выгодский М.Я. Справочник по элементарной математике. Издание 17-е

стереотипное. – М.: Наука, ГРФМЛ, 1966. – 424 с.

9. Данко П.Е., Попов А.Г., Кожевников Т.Я. Высшая математика в

упражнениях и задачах. Ч.1. Издание 4-е, испр. и доп. – М.: Высшая школа,

1986. – 304 с.

10. Кудрявцев Л.Д. Математический анализ. Т.1. – М.: Наука, ГРФМЛ, 1970.

– 590 с. » [8, 9].

28


2.2 Содержание элективного курса «Раскрытие неопределенностей

при вычислении пределов»

1. Введение.

Значительное место в школьном курсе математики занимают элементы

математического анализа, в том числе и пределы функций с раскрытием

неопределенностей. Целью изучения в школьной программе этой темы

является формирование интеллектуального развития учащихся,

формирование качеств мышления, необходимых человеку для свободной

ориентации в современном мире; овладение математическими знаниями,

необходимыми для применения в практической деятельности, для изучения

смежных дисциплин, для продолжения образования. Но как показывает опыт

преподавания учителей в школе, вычисление пределов вызывает большие

затруднения у школьников по сравнению с другими темами. В разделе

«Предел функции и непрерывность» заметен высокий уровень научности и

строгости понятий предела и непрерывности функции. Раскрытие

неопределенностей – методы вычисления пределов функций, заданных

формулами, которые теряют смысл в результате формальной подстановки в

них предельных значений аргумента, то есть переходят в выражения

; 0 ; ∞ − ∞; 0 ∙ ∞; ∞ 0 00 ; ∞ 0 ; 1 ∞ .

Вопрос решения пределов является достаточно обширным и является

объектом интереса современных направлений математики. Существуют

десятки нюансов и хитростей, позволяющих решить данный предел.

Объектом нашего исследования правила раскрытия неопределенностей и

правила Лопиталя. Можно привести огромный список литературы, в которой

изучаются пределы, способы их вычислений. Вместе с тем, при изучении

нами различных публикаций по данной тематике выявлена относительная

недостаточность данных в курсе школьной математики. В основном

материалы представлены для изучения в высших учебных заведениях. В

курсе же 10 класса отводится всего лишь 10 часов на раздел. Поэтому

предлагаю методические рекомендации по методике раскрытия

неопределенностей ; 0 ; ∞ − ∞; 0 ∙ ∞; ∞ 0 00 ; ∞ 0 ; 1 ∞ при вычислении

пределов функции.

Способ раскрытия такого рода неопределённостей был опубликован

Лопиталем в его сочинении «Анализ бесконечно малых», изданном в году. В

предисловии к этому сочинению Лопиталь указывает, что без всякого

стеснения пользовался открытиями Лейбница и братьев Бернулли и «не

имеет ничего против того, чтобы они предъявили свои авторские права на

все, что им угодно». Иоганн Бернулли предъявил претензии на все сочинение

Лопиталя целиком и в частности после смерти Лопиталя опубликовал работу

под примечательным названием «Усовершенствование моего

опубликованного в “Анализе бесконечно малых” метода для определения

значения дроби, числитель и знаменатель которой иногда исчезают» [ 9, 10].

29


2. Последовательность как функция натурального аргумента.

Предел числовой последовательности.

Бесконечной числовой последовательностью называется числовая

функция, определенная на множество N натуральных чисел.

Обозначим эту функцию, например, буквой f. Последовательность

общим членом обозначают:

f n

1 2

f , f , f n

,

Наиболее употребительные способы задания последовательности

следующие:

1. Аналитический способ. При этом способе указана формула,

позволяющая по номеру вычислить соответствующий член

последовательности.

1

Пример. xn

1 n

N 2

n

1 1 33

По этой формуле x5 1 1

5

2 32 32

В этом случае говорят, что последовательность

n

x

n

задана формулой

1

xn

1 , n N или просто

n

2

2. Рекуррентный способ. При этом способе задают первый член

последовательности и формулу, позволяющую определить любой член

последовательности по известным предшествующим (одному или

нескольким).

Пример

при любом

позволяют найти каждый член (начиная со второго)

an

an

1

d

a a d

n

n1

последовательности

a

n

1

xn

1

2

n

n 2 .

через предыдущий член соответствующей

последовательности. Это арифметическая прогрессия.

3. Описательный способ. При этом способе дается описание того,

каковы элементы последовательности. Причем очень часто ни формулы для

общего члена, ни рекуррентного соотношения для членов этой

последовательности нет. Для иллюстрации укажем последовательности

2, 3, 5, 7,...,- последовательность простых чисел.

Последовательности являются функциями, определенными на

множестве N, поэтому для них имеют некоторые из простейших

характеристик произвольных числовых функций, также как ограниченность

и неограниченность, несколько видоизмененная периодичность,

монотонность.

Число е называется пределом последовательность

x n , если для

любого положительного числа существует такое натуральное число n

, что

для всех xn

a . При этом пишут: lim x n

a или xn

a при n [11,

12, 13].

n

30


Последовательность, у которой существует предел, называется

сходящейся.

Последовательность, не являющаяся сходящейся, называется

расходящейся.

Теорема. Если последовательность сходится, то она имеет только один

предел.

3. Предел функции.

Определение. Число А называется пределом функции при

стремлении х к а (или в точке а), если для любой сходящейся к а

последовательности , где

для , соответсвующая

последовательность

x

n

f x

n

Используя обозначение:

A lim f x или

x a

Символически:

f x

xn

31

a

значений функции

A

при x a

.

n

n n n

f

n

N

A lim f x x ; x a x a f x A,

n

N

xa

иcходится к числу А .

Определение. Число А называется пределом функции при стремлении х

к а , если для любого положительного число существует зависящее от

положителное число , что для всех х, удовлетвлояющих неравенсту

, справедливо неравенство

. Обозначение прежнее:

0 xa

A lim f x

x a

Символически:

f x A

0

A lim f x 0 0 xU

a f x U

A

xa

Пример. Пусть

f x

2

x 2x1

2x

2

Возьмем произвольную последовательность

n

1

x n N

. Выясним существует

x

n

такую, что

f

1

lim f x

x

lim x 1

. Кроме того, считаем xn

1, так как f не определена при

x 1 .

Используя теоремы о последовательностях, имеем

lim

x

2

x x xn

n

2

2 1 lim 2lim

n

1

n

x

x

x

12 1 2 1

2x

2 2lim x 2 2 2 4 2

n

x

Следовательно, существует lim f x

n

x1

зависит от выбор последовательности

существует и

lim f x

x1

1

.

2

x1

1

n

. Причем этот предел не

2

x с указанными свойства. Поэтому

n

n

.

,


Определение. Число А называется пределом функции при

стремлении х к положительной (отрицательной) бесконечности , если для

любой бесконечно большой последовательности значений аргумента,

x

n

члены которой, начиная с некоторого номера, положительны (отрицательны),

соответсвующая последовательность

иcходится к числу А .

Используя обозначение:

A lim f x

x

A lim f x

x

f x

n

значений функции f

Определение. Число А называется пределом функции при

стремлении х к положительной (отрицательной) бесконечности, если для

любого положительного число существует такое число с , что для всех х,

удовлетвлояющих неравенсту

, выполняется неравенство

f x A

.

Символически:

x c x c

f

f

32

.

[14, 15].

4. Бесконечно малые и бесконечно большие функции.

Здесь снова все рассматриваемые функции считаются определенными

на некотором интервале (c,d) (конечном или бесконечном) за исключением,

быть может точки Если а=с или а=d, то понимается соответственно

предел справа или слева. Не исключается и случай a или

Определение. Функция

называется бесконечно малой

функцией (или просто бесконечно малой) при если lim ( x) 0.

a( c, d).

( x)

x

a,

xa

a .

С помощью понятия бесконечно малой функции можно

сформулировать (также как и для последовательностей) еще одно

определение предела функции, содержащееся в следующей теореме.

Теорема. Для того, чтобы выполнялось равенство

lim f ( x ) A

x a

необходимо и достаточно, чтобы

f ( x) A ( x),

где lim ( ) 0.

xa

x

(Символически: lim f ( x) A f ( x) A ( x), lim ( x) 0. ).

xa xa

Справедливы следующие свойства бесконечных малых.

Лемма 1. Алгебраическая сумма любого конечного числа бесконечно

малых при x a функций есть функция бесконечно малая при x a.

Лемма 2. Произведение ( x) f ( x)

бесконечно малой при x

a

функции ( x)

и функции f( x ), ограниченной в окрестности точки a ,

является функцией бесконечно малой при x

a.


Следствие. Произведение конечного числа Функций, бесконечно

малых при x a, есть функция бесконечно малая при x

a.

Наряду с понятием бесконечно малой функции часто используется и

понятие бесконечно большой (при или при

)

функции. Сформулируем одно из определений бесконечно большой при

x a функции.

Определение. Функция называется бесконечно большой при ,

если для любого числа существует число

такое, что для

всех удовлетворяющих условию

следует выполнение

x,

неравенства

f ( x) E.

E 0

f

x

a

В этом случае пишут

0 xa

( E) 0

lim f ( x ) .

a

x

xa 0, ха

0

Аналогично формулируются определения функции бесконечно

большой определенного знака слева или справа в точке a . Обозначения

таковы:

lim f ( x) ; lim f ( x) ; lim f ( x) ; lim f ( x) .

xa0 xa0 xa0 xa0

Отметим простую связь между бесконечно малыми и бесконечно

большими функциями.

Теорема. Если - бесконечно малая при

x

a,

то

1

f( x)

( x)

( x)

- бесконечно большая при

( x)

x

a.

x a ( x) 0

x

a

Доказательство. Функция бесконечно малая при По

определению бесконечно малой для любого

что из неравенства

следует неравенство

( ) 0,

0 ( x) .

0 xa

,

при

x

a

Отсюда

т. е.

0 xa

x

a.

при

0 существует такое

1 1

E для всех x , удовлетворяющих условию

( x)

1

lim .

xa( x)

Аналогично доказывается утверждение: если

x

a,

то функция

x

a.

f

а, определяемая выражением

бесконечно большая

1

( x) ,

f( x)

бесконечно малая при

Сравнение бесконечно малых и бесконечно больших функций.

Как установлено выше, сумма и произведение любого конечного числа

бесконечно малых при x a функций являются бесконечно малыми при

функциями. Однако частное двух бесконечно малых функций

оказывается необязательно функция бесконечно малая.

Следующие простые примеры показывают возможные для такого

отношения случаи:

( x)

( x)

2

1. ( x) x , ( x) x lim lim x 0;

x0 x0

33


( x) 1

( x)

x

2

2. ( x) x, ( x) x lim lim ;

x0 x0

( x)

( x)

2 3 2

3. ( x) 2 x x , ( x) x lim lim(2 x) 2;

x0 x0

( x)

( x)

2 3 2

4. ( x) x 2 x , ( x) x lim lim(1 2 x) 1;

x0 x0

1 ( x) 1

5. ( x) x cos , ( x) x lim limcos

x x0 ( x)

x0

x

- не существует.

Для сравнения бесконечно малых как раз и используется характер

поведения их отношения при Пусть и бесконечно малые

при x a функции при

при

Определение 1. Функция называется бесконечно малой более

высокого порядка, чем при x

a, если

( x)

lim 0.

xa

( x)

x

a.

( x) 0, ( x) 0

( x)

( x)

( x) o( )

Используется обозначение:

есть o малое от при x a ).

Определение 2. Функция и

(при x a ) одного порядка, если

( x)

lim C 0,( C const).

xa

( x)

Определение 3. Функция и

бесконечно малыми (при x a ), если

( x)

lim 1

xa

( x)

или

( x)

( x)

lim 1.

xa( x)

x

( x)

( x)

( x)

( x)

x

a.

( x)

при x a

(читается: функция

называются бесконечно малыми

называются эквивалентными

Используется обозначение: ~ при x a (читается функция

эквивалентна при x a).

Если предел отношения двух бесконечно малых (при x a ) не

существует, то говорят, что эти бесконечно малые несравнимы между собой.

В рассмотренных примерах:

или при x 0;

1) o( )

2) o( )

или

3) ( x)

4) x

x

x

x

2

2

o( x)

o( x)

x 0

при

x

x 0;

и ( x)

- одного порядка при x 0 или 2x

бесконечно малые одного порядка при x 0;

~ при или

при x 0;

при x 0 несравнимы между собой или

собой при x 0.

2x x x

2 3 2

2 3

x и

2

x -

5) ( x)

и ( x)

1

x cos x и x несравнимы между

x

34


Следующая простая теорема дает необходимое и достаточное условие

эквивалентности двух функций.

Теорема 1. Для того, чтобы две функции

при были эквивалентными при необходимо и

достаточно, чтобы выполнялось хотя бы одно из условий:

или

( х) 0

о( )

х

Следствие. Пусть

C o( )

при

Теорема 2. Пусть

1( x)

lim , то существует и

xa

( x)

1

x

a.

( x) ( x)

1

lim lim .

x a ( x) x a 1( x)

о( ).

( x)

lim C

0,

xa

( x)

1

и

1

( x)

lim ,

xa

( x)

35

( х), ( х) 0,

х

а,

( х),

где C -постоянная, тогда ~С и

при

причем

х

а.

Тогда, если существует

Теорема 2 позволяет при вычислении пределов отношения двух

бесконечно малых заменять каждую из них любой эквивалентной ей

бесконечно малой, причем это не сказывается на значении предела

х

а,

отношения при если он существует. Если один из пределов

( x) 1( x)

lim lim не существует, то не существует и другой.

x a ( x) x a 1( x)

Следовательно при вычислении пределов отношений бесконечно малых эти

бесконечно малые можно заменять им эквивалентным, не заботясь о

предварительном выяснении вопроса о существовании пределов.

В последующем материале будет обоснована при х→0 эквивалентность

бесконечно малых:

x

x ~sin x ~tgx ~arcsin x ~arctgx ~ln(1 ) ~

Рассмотрим примеры.

x

e 1

2 2 2 2 2

1 cos2x tg x 2sin x tg x 2x x

1. lim lim lim 3,

x0 0 0

2

x sin

x x x sin

x x

x

здесь использована формула

, sin x ~ x , при x 0.

tgx ~ x

1cos2x2sin

arcsin( x2) x2 1 1

2. lim lim lim ,

x 2 2

x 2 x x 2 x( x 2) x 2

x 2

2

x

и эквивалентность:

здесь использована эквивалентность: arcsin( x 2) ~ при x 2.

При одновременном рассмотрении нескольких бесконечно малых одну

из них или некоторую другую берут в качестве стандартной (или основной) и

сравнивают с ней все другие.

x 2

Например, для бесконечно малых при x a функций в качестве

k

стандартной берут функцию ( x a) . На этом основан метод выделения

главной части функции.


( x) и ( x)

Определение. Если

- бесконечно малые при и

( x) ( x) o( ),

то бесконечно малая называется главной частью

бесконечно малой .

Можно доказать, что если для бесконечно малой существует

( x)

x

a

главная часть вида

то она единственная.

Достаточно общий способ выделения главной части функции даст так

называемая формула Тейлора.

Метод выделения главной части функции позволяет данную

сравнительно сложную бесконечно малую функцию в окрестности данной

точки заменить с точностью до бесконечно малых более высокого порядка

другой более простой бесконечно малой (например, степенной ).

Бесконечно большие функции классифицируются также по характеру

поведения их отношения в окрестности рассматриваемой точки. [16, 17].

Пусть - бесконечно большие при х а функции.

Определение 1. Функция называется бесконечно большой более

высокого порядка, чем g при если

f

и g

f( x)

lim или

xa

gx ( )

k

A( x a) ,

f

x

a,

gx ( )

lim 0.

xa

f( x)

Определение 2. Функции

одного порядка при если

f( x)

lim С

0,

xa

gx ( )

x

a,

Пример. Функции

f

и g

где С-постоянная.

1

x

f( x)

2

x

и

A( x a) k

называются бесконечно большими

gx ( )

1

2

x

имеют одинаковый порядок

роста при х→0, так как

f( x)

lim lim(1 х) 1.

x0 gx ( ) x0

При одновременном расмотрении нескольких бесконечно больших

одну из них берут в качестве стандартной и сравнивают с ней остальные.

Рассмотрим примеры.

2 2

2

x 3x x 3x 3x

3

1. lim ( x 3 x x) lim lim lim ,

x x 2 x 2

x

x 3x x x x 2x

2

здесь после перевода иррациональности в знаменатель проведена при x

замена

x

2

3x

главной частью

2

x x.

6 4 3 9 6 3 9 3

x x 1 3 x 1 x 3 x 4x

2

2. lim lim lim .

x 3 6 3 3 6

3

4x 3 x 4x 2x 6 x

4x 2 x

x

6x

3

Здесь использовано то, что главной частью каждого многочлена при

x является старший член его.

36


x 3.

3 3

x 1

x

1.lim lim lim .

x 2

2 x

x 1

x x x

2x 3 2x

1

2.lim lim 2lim 0.

x

2

2

x 1

x x x x

Пример. Найти левый и правый пределы функции

Решение:

Следовательно,

1/ ( x 3) , 2 x

x

a.

x

a

Если

x

3 0

x 30,

lim f( x) 1/ 3.

1/( 3)

и

x

3 0

37

то

lim f( x) 0.

1/ ( x 3)

Если же

Пример. Найти левый и правый пределы функции

Решение: Если

0,

x

то 1/ ( )

a

0,

x a и

x

a

то 1/ ( )

lim f( x) .

xa0

и

xa0

f( x)

x

и

1

1/( 3)

2 x

x 30,

f ( x)

e

1/( xa)

lim f( x) 0.

при

1/( x3)

2 0.

при

Если же

Пример. Пусть t бесконечно малая величина. Сравнить бесконечно

2 5

малые 5t

2t

и

Решение: Найдем

2 3

3t

2 t .

2 5 2

5t 2t 5 2t

5

lim lim lim .

t 0 t 0

2 3

3t 2t t 0

3 2t

3

Так как предел отношения к есть число, отличное от нуля, то и

—бесконечно малые одного и того же порядка .

при

при

Пример. Сравнить бесконечно малые величины

t 0.

Решение: Находим

2

tsin t 1

lim lim limsin 0,

t 0 t 0 2tsint

2 t

0

т. е. 0( ).

Пример. Сравнить бесконечно малые величины

t 0.

Решение: Имеем

ln(1 t)

t ln(1 t) ln(1 t)

lim lim lim lim t 1,

t 0 t 0 t sint t 0 sint

t

0 sin t

t

2

a t sin t

tln(1 t),

т. e. ~

то

и 2tsin

t

tsint

ln(1 3xsin x)

Пример. Найти lim .

x0

2

tgx

Решение: Заменим числитель и знаменатель дроби эквивалентными

2 2

бесконечными малыми: ln(1 3xsin x) ~ 3xsin x, tgx ~ x . Тогда получим


ln(1 3xsin x) 3xsin x sin x

lim lim 3lim 3. [18, 19].

x0 2

x 0

2

tgx x x0

x

5. Непосредственное применение теорем о пределах. Раскрытие

неопределенностей.

Эти теоремы относятся к арифметическим операциям над функциями.

Теорема 1. Если существуют lim ( ) и lim ( ) , то существует

lim f ( x)

g x

xa

, причем

f x A

x a

g x B

x a

f x gx f x gx

lim ( ) lim lim

xa xa xa

Теорема 2. Если существуют lim ( )

lim f ( x)

g x

xa

, причем

Замечание. Если

x

a f x

и lim ( )

x

a gx

f x gx f x

gx

lim ( ) lim lim

xa xa xa

f ( x)

c

Следстивие 1. Если существует

lim с f ( x) c lim f ( x)

xa xa

Следстивие 2. Если существует

k

f x f x

lim ( ) lim ( ) k

xa xa

f( x)

lim

x a g x

f x c

x a

(постоянная), то lim ( )

lim f ( x )

a

x

lim f ( x )

a

x

Теорема 3. Если существуют lim ( )

, причем

x

a f x

f( x)

lim f x

xa

lim

xa

g x

lim g x

xa

.

, то существует

.

.

, то для любого числа с

, и k-натуральное число, то с

и lim ( ) 0

x

a gx

, то существует

Нужно иметь в виду, что знак ∞ - это только символ для обозначения

бесконечно большой величины. Он не обладает свойствами числа и в

арифметических действиях не участвует. Вследствие этого возникают

различного рода неопределённости. Основные виды неопределенностей:

∞ ± ∞,

∞ , 0

0 , 0 ∙ ∞, ∞0 , 0 0 , 1 ∞ .

Вычисление пределов в этих случаях называют «раскрытием

неопределенности». Вышеуказанные теоремы для бесконечных пределов

неверны. Для вычисления предела – «раскрытие неопределенностей»,

предварительно преобразовывают выражения.

Найти пределы:1) lim (3x2

6x

5)

; 2)

x1

;

4 lim lgx

x1

4

5

lim ;

x2

x 2

3

3

6 lim ;

x

tgx

2

x

x lim 7 5

5

10 2x

x2

7 lim ;

x0

1/ x2

e

;

x2

lim 3x

;

3

x2

38


Решение: 1) Используя теоремы:

lim (3x2

6x

5) lim (3x2 ) lim (6x)

3 2 6 5

1

1

1

lim5 1

lim x

1

lim x

x

x x x x x1

3(

lim x)(

lim x)

61

5 3 1

2

x1

x1

2)

3)

4)

5)

6)

7)

lim 7x

5

7x

5

lim x 5

x510

2x

lim 10 2x

x 5

lim 3 6

x2

x

и

lim 2 4

x2

x

lim

lim

lg x lg x lg1

0.

x1

x1

lim 2

3

x2

x

0

,

lim 7x

lim 5 7 lim x 5

x 5 x 5 x 5

lim 10 lim 2x

10 2 lim x

x 5 x 5 x 5

,

4

lim

2 x 2

x

lim 2

x 3

lim tgx tg

lim x

x

x

2 2

lim 2 0

x0

x

и

lim 1/ 2

e x

x0

Пример. Найти

,

,

2

lim .

x4

3

x2

3x 1296

.

3

lim 0.

x

tgx

2

lim x2

0

x0

1/ x2

e

.

.

755 30 3 .

10 25 20 2

Решение: Так как х 4, то числитель дроби стремится к числу

а знаменатель - к числу

Следовательно,

54 2 22,

2 22

lim 2.

x4

3 11

при

2 4 3 11.

5

Пример. Найти lim .

x

7

Решение : Числитель и знаменатель дроби неограниченно возрастают

х . В таком случае говорят, что имеет место неопределенность вида

. Разделив на х

3х5 3 5 х 3

lim lim ,

x

2х7 x

2 7 х 2

числитель и знаменатель дроби, получаем

так как при х каждая из дробей 5 х и 7 х стремится к нулю.

2

х 9

Пример. Найти lim .

x3

2

х 3х

Решение: Здесь числитель и знаменатель дроби при х 3 стремятся к

нулю (принято говорить, что получается неопределенность вида 0/0). Имеем

2

х 9 ( х 3)( х 3) х 3

2

;

х 3 х х( х 3)

х

39


Если

х 3, то

стремится к числу

Пример. Найти

2

х 9 х3

lim lim . Но при

x3 2

х 3х x3

х

3

3 2.

3

Итак,

3 2

х х х 1

lim .

x1

3 2

х х х 1

2

х 9

lim 2.

x3

2

х 3х

х 3 дробь

Решение: Здесь имеет место неопределенность вида 0/0. Разложим на

множители числитель и знаменатель дроби:

3 2 2

х х х 1 x ( x 1) ( x 1)

lim

lim

x1 3 2

1

2

х х х 1 x

x ( x 1) ( x 1)

2

( x 1) ( x 1) x 1 0

lim lim 0.

x1 2

( x 1)( x 1) x1

x 1 2

Пример. Найти

3

х 1000

lim .

x10

3 2

х 20х 100х

Решение: Это - также неопределенность вида 0/0. Имеем

3 2 2

х 1000 ( x 10)( x 10x 100) x 10x

100

lim lim lim .

х 20х 100 х x( x 10) x( x 10)

x 10 3 2 x 10 2

x 10

Числитель дроби стремится к 300, а знаменатель стремится к нулю, т. е.

является бесконечно малой величиной, следовательно, рассматриваемая

дробь — бесконечно большая величина и

Пример. Найти

x 42

lim .

x0

x

3

х 1000

lim .

x10

3 2

х 20х 100х

Решение: Умножим числитель и знаменатель дроби на сумму

x 4 2:

( x 4 2)( x 4 2) x 4 4 1 1

lim lim lim .

x 0 x( x 4 2) x 0 ( x 4 2) x 0

x 4 2 4

Пример. Найти

Решение:Положим

5 3

(1 x) 1

lim .

x0

x

5

1 x

y ,

тогда

5 3 3 2

(1 x) 1 y 1 y y 1 3

lim lim lim .

x 0 y 1

5

1

4 3 2

x y 1 y y y y y 1 5

Пример. Найти

3 2

х 2х 3х

4

x

3 2

4х 3х 2х

1

lim .

/ .

Решение: Это - неопределенность вида

знаменатель дроби на старшую степень x , т. е. на x

3 :

х

х

3

Разделим числитель и

40


3 2 2 3

х 2х 3х 4 1 2 / x 3 / x 4 / x 1

lim lim .

x

3 2 2 3

4х 3х 2х 1 x

4 3 / x 2 / x 1/ x 4

Пример. Найти

4

3x

2

lim .

x

8

x 3x4

Решение: Разделим числитель и знаменатель на

4 4

3x

2 3 2 / x 3

lim lim 3.

x

8 x

7 8

x 3x 4 1 3 / x 4 / x 1

Пример. Найти предел:

41

x 4 :

lim 2 x 8x 3 2

x 4x

3.

x

Решение: Здесь имеет место неопределенность вида

разделим данное выражение на

lim

x

lim( 2 x 8x 3 2

x 4x

3)

x

2 2

x x x x

8 3 4 3 :

2 2 2 2

x x x x x x x x

8 3 4 3 8 3 4 3

2 2

x x x x

8 3 4 3

2 2

x 8x 3 x 4x 3 4x

lim

lim

x

2 2 x

2 2

x 8x 3 x 4x 3 x 8x 3 x 4x

3

4 4

lim 2.

x

2 2

1 8 / x 3 / x x 4 / x 3 / x 2

Пример. Найти предел:

3

1

x 2x

lim

x

3 2

x x 1

. Умножим и

Решение: Теорему о пределе частного применять нельзя, так как

числитель и знаменатель дроби конечного предела не имеют. Имеем

неопределенность вида ∞ . Для избавления от неопределенности вынесем за

скобки в числителе и знаменателе дроби переменную в старшей степени:

1 1

3

2

1

x 2x

3 2 0 0 2

lim

lim x x

2

x

x3

x2

1

x

1 1

1

1

0 0

x x3

lim

x

Пример. Найти предел: x2

4x

1

x

lim

x

3

x3

5 3

Решение:

4 1

1

1

x2

4x

1

x

x 2 11

lim

x

2.

3 3

5 3 1

0

5 3 x

x

31

x3

x

Пример. Найти предел: lim x2

x 2

.

x2

3x2

4x

4

.


Решение: При вычислении данного предела применять теорему о

пределе частного нельзя, так и числитель, и знаменатель равны 0.

Воспользуемся разложением многочленов числителя и знаменателя на

множители по формуле

где x 1 , x 2 - корни

квадратного трехчлена.

х

2

a ( x x

ax 2 bx c

)

2

ax bx c

) ( x x

ax 2 bx c

)

х 2 0

,

x

2 4х 4 0

x

lim

x2

3x

2

2

1,2

,

b

x

1,2

1

2

b

2a

b

4ac

2

b

2a

2

Тогда

1

4ac

42

a ( x x

1 4 2

2a

4

x

1 x 2

x 2 0

lim

4x

4 0 x2

2

3 x

3

x 2

1

) ( x x

1 3

;

2

2

х

16 48 4 8

,

2 3 6

1

1;

х

1

х

2

2

3

x 1 2 1

lim

x2

2 2

3 x 3 2

3 3

2

;

3

8

;

х 2 2 ;

8 x 3

Пример. Найти предел: lim

1 2

.

x

x 1

Решение: Числитель и знаменатель дроби при х→0 стремятся к нулю,

следовательно, имеем неопределенность вида 0 . Тогда

0

8 x 3 0

lim lim

x1

2

x 1

0 x1

x

1

lim

x1

2

lim

x16

4

8 x 3

8 x 3

lim

2

2

x

1

8 x 3 x1

x

1

8 x 3

lim

x

1 x

1 8 x 3 x1

x

1 8 x 3 2 8 1

3

Пример. Найти предел:

Решение:

4

x 0 2 t

lim

0 2

2

x

t

4 t

Пример. Найти предел:

2

lim

x16

4

lim

t2

4

x

x

1

.

8 x 9

4

2 t

2 t 2 t

8 x 3

lim

x 1 2

x 1

.

lim

t2

1

2 t

1

1

.

1

.

12

Решение: При подставлении предельного значения 1 вместо x в данном

выраженим получим неопределенность 0 .

0

Для вычисления предела функций с иррациональными выражениями,

часто применяем следующие утверждения:

a) перевод иррациональности с числителя в знаменатель и наоборот;

б) перевод иррационального выражения в рациональное путем

введения новой переменной.

Для вычисления данного предела сначала переведем иррациональность

с числителя в знаменатель (для этого умножаем и числитель и знаменатель

.


на сопряженное выражение числителя 8 x 3), затем сокращаем дробь

на множитель х1, и находим:

lim

x1

lim

x1

8 x 3 0

lim

2

1

1

0

x

x

x 1

8 x 3

8 x 3

8 x 9

lim

2

x1

2

x

1

8 x 3 x

1

8 x 3

lim

x1

x

1 x

1 8 x 3 x

1 8 x 3 2 8 1

3

6. Два замечательных предела.

1. Первый замечательный предел.

Теорема 1.

sin x

lim 1

x0

x

1

(1-замечательный предел).

1

1

.

12

Доказательство. Из рис.1, сравнивая площади треугольников

OAB,

OAC с площадью сектора

неравенства

sin x x tgx

x

При делении на sin 0

x 1

1 .

sin x cos x

OAB (| OA | 1),

из sin x x tgx

получим при 0

следует

Для обратных величин справедливы обратные неравенства

sin x

1

cos x.

x

Умножением неравенства

sin x

1

cos

x

x

x

2

[20].

на число (-1) и добавлением

числа 1 к каждой части их, получим

sin x

0 1 1

cos x.

x

sin x

Заметим, что неравенства 0 1 1 cos x справедливы при

x

0 x

. Далее при указанных х использованием правой части неравенства

2

2 x x x

получим, что 1 cos x 2sin 2sin 2 x,

2 2 2

следовательно,

sin x

0 1

1

cos x

x

sin x

0 1 x, 0 x

.

x

2

Функции 0 и х при х 0 имеют один и тот же предел (число 0),

sin x

поэтому из 0 1 1 cos x следует , что

x

43


sin x

lim1

0

x0

x

или

sin x

lim 1.

x

x0

Вследствие нечетности синуса и замены t x

sin x sin( x) sint

lim lim lim 1

x ( x)

t

x0 x0 t0

или

Так как оба односторонних предела

sin( x)

lim 1

x

x0

sin x

lim 1

x

x0

и

sin( x)

lim 1

x

x0

то существует двухсторонний предел, равный тому же числу, т.е.

sin x

lim 1.

x0

x

2. Второй замечательный предел

Теорема.

lim(1 x

x0

1

) x

e

Пример . Найти предел:

(второй замечательный предел)

sin mx

lim .

x0

x

Решение: Используя первый замечательный предел, имеем

sin mx m sin mx sin mx

lim lim m lim m.

x0 x x0 mx x0

mx

Пример . Найти предел:

Решение: Имеем

1

cos5x

lim .

x0

2

x

2

2 2

1 cos5x 2sin (5 x / 2) sin(5 x / 2) 5 25

lim lim 2lim 2 .

x0 2

x 0

2

x x x0

x 2

2

Здесь мы воспользовались результатом предыдущего примера, приняв

m 5 / 2.

равны,

2sin2x

Пример . Найти предел: lim

x→0 5x

Решение: Для избавления неопределенности 0 , воспользуемся первым

0

замечательным пределом

2sin2x 2 ∙ 2sin2x

lim = lim

x→0 5x x→0 2 ∙ 5x

Пример . Найти предел: lim

lim

x→0

arcsin3x

x→0 tg7x

= lim

x→0

( 4 5 ∙ sin2x

2x ) = 4 5

Решение: Произведя следующие преобразования, имеем

arcsin3x 21x ∙ arcsin3x

= lim

= lim ( 3 tg7x x→0 21x ∙ tg7x x→0 7 ∙ arcsin3x 7x

3x

Пример . Найти предел: lim

x→0

1−cosx

3x 2

Решение: Так как 1 − cosx = 2sin 2 x 2 , то

tg7x ) = 3 7 ∙ 1 ∙ 1 = 3 7

44


1 − cosx 2sin 2 x

2sin

lim

x→0 3x 2 = lim

2

2 x

x→0 3x 2 = lim

2

= 1

x→0

4 ∙ 3 x2 6 lim ( sin x 2

x→0

x ) = 1 6 ∙ 12 = 1 6

4

2

Пример . Найти предел: lim(1 − x)tg πx

x→1 2

Решение: Неопределенность вида 0 ∙ ∞ сведем к неопределенности 0 , 0

тогда

lim

x→1

1 − x

ctg πx = ( 0 0 )

x→1

2

Сделаем замену переменных z = 1 − x, тогда z → 0, ctg πx

πx

(1 − x)tg = (0 ∙ ∞) = lim

2

ctg π(1−z)

2

= tg πz

2 и

1 − x

lim

x→1

ctg πx = lim

z→0

2

z

tg πz

2

= lim

2 ∙ 2 π

tg πz

2

z→0

πz

= 2 πz

π ∙ lim 2

z→0

tg πz

2

45

2

= 2 π ∙ 1 = 2 π

2 =

Пример . Найти предел: lim (1 + 2

x→∞ x )x

Решение: Сведем выражение ко второму замечательному пределу

lim (1 + 2

x→∞ x )x = lim (1 + 2

x→∞ x )2x∙12 = ( lim (1 + 2

x→∞ x )2x )

1

2

= e

1

2 = √e.

Пример . Найти предел: lim (1 + 1

x→∞ x )2x

Решение: Сведем выражение ко второму замечательному пределу

lim (1 + 1 2x

x→∞ x )

x

lim 0

x

lim 0

t

lim 0

= lim (1 + 1 x∙2

x→∞ x ) = ( lim (1 + 1 x

x→∞ x ) )

Пример. Найти предел:

Решение:

sin 2x

sin 3x

0

0

x

lim 0

Пример. Найти предел:

Решение:

sin 2x

lim

x 0

sin 3x

.

2

= e 2

sin 2x

lim sin 2x

2x

2x

0 2

lim 2 1

x

x x

sin 3x

lim sin 3x

0 3 1

3

x

x

x

3x

x0

3x

arcsin 2x

lim

x 0 x

sin t

arcsin 2x

0 arcsin 2x

t 2x

sin t x

2

x 0 x 0 t 0

t

sin t

2

2

t

lim 0

t

21

2 . [22, 23].

sin t

.

2

3

2

3

.


lim

x

lim

x

Пример. Найти предел:

Решение:

x 1

lim x

x 1

x

x

x

x 1

lim

x

x 1

x x 1

x

x 1

2

1

x 1

Здесь,

lim

x

x1

2

x

2 x1

x

.

x 1

2

1

lim 1

1

lim 1

lim

x

2

1

x 1

2x

lim

x 1

x

Пример. Найти предел:

Решение:

2

1

x 1

x1

2

2x

x1

x

2 1

lim

2 0

x1

2

2

1

1

x

lim

x 0

.

1

1

e

x

x 1

. Тогда, lim e2

. [21].

x

x 1

tgx

x

lim tgx

lim sin x

(

x0

x x0

x

1

cos

lim sin x

)

x x0

x x

lim 0

1 1

1

1.

cos x 1

Пример. Найти предел: lim

5x

.

x 0sin4x

Решение: lim 5x

5/ 4 5/ 4 5/ 4 5

lim 1,25.

x0

sin 4x

x0

sin 4x

lim sin 4x

1 4

4x

x0

4x

Пример. Найти предел:

Решение:

x

x

lim 1 cos

0

x

.

2sin 2 x x

sin

lim 1

cos x

lim 2 lim 2 lim x

sin 1

0 0.

x0

x x0

x x0

x / 2 x0

2

Пример. Найти предел:

Решение:

1

lim xsin

x

x

1

sin

1

lim xsin

lim x

x

x x

1

x

.

1

y

lim sin y

x

1.

,

0

y0

y

x y

nx

Пример. Найти пределы:

cos x cos3x

cos

1) lim

; 2) lim 2 .

x0

x2

x1

1

x

Решение: 1) cosx

cos3x

4sin 2 xcosx. Тогда,

2

lim cos x cos3x

lim 4sin 2 xcos

x

lim sin x

4 lim cos x 4.

x0

x2

x0

x2

x0

x x0

и

46


2)

nx

cos

cos t t

1 2 2

sin

lim 2 x t

lim lim 2

x1 1

x x1 t

0 t0 t t0

t

t

sin

lim 2

1 .

2 t0

t

2 2

2

Пример. Найти пределы:

x

2

1

x2

1 x2

1) lim ( )

1

lim (1 )

1

lim (1

x

x

2 x

x

2 x

x

1

x2

1

lim (1 ) lim (1 ) e 1

e

x

x

2 x

x

2

2) lim (1

3

)

x

x

lim (1

1

)

x

x

x / 3

x / 3

lim

x

1

(1 )

x /3

eee

e

3

.

3

x / 3

1

lim 1

x

x /3

1

2

x / 3

x2

1

) (1

)

x

2

1

lim 1

x

x /3

3) lim (

x1

) x lim (1

x1

1)

x lim (1

4

) x

x

x3

x

x3

x

x3

lim 4

4 4 lim 4

1

x

1 3

4 3 x

lim [(1 ) ] x x

3 lim 3 x x

e x x

e x

3

e x e4 ,

x

x3

x

4

x

1

4 3

lim (1 ) e

x

x 3

5

4) lim (1 sin x)

x

x0

e

51

e,

.

x / 3

5sin 5 lim sin

sin 5 lim

) sin1 ]

0 x x

lim [(1 sin

0 x x

x

x

x x e x e x

x0

1

здесь lim (1 sin x)

sin x e ( x 0 sin x 0).

x 0

Пример. Найти предел: tgx ctgx

x

lim 0

1 .

lim

x0

Решение:

ctgx

ctgx

1

tgx

:

1/ tgx

1

tgx lim 1

tgx x 0 да lim 1

a e.

x0

Пример. Найти предел: ln1

x

lim .

x0

x

tgx a

a 0

a0

1

a

47


Решение:

lim ln 1

x

x0

x

ln 1

x 1

1

ln 1

x ln 1

x

x x

lim ln 1

x0

Здесь ln1

x

x

lim 0

x 1/ x

x

ln lim 1

x

x

1.

Пример. Найти предел:

Решение: Замена

x a

a x 1

z x ln a ln 1

z

a 1

lim

x z

lim

x0

x z0 ln 1

Сонымен

x0

a x

x

lim 1

0

x

1

z

. Тогда

ln 1

z

x

ln a

lim

z0

ln a

.

.

1/ x

lne 1.

x 0,

z 0

1

ln a

lim

z 0

z ln1

z ln1

z

a 1

lim

x

ln a.

x0

x

Пример. Найти предел:

Решение:

lim

x

x

x

2

x

3

z

x

x 2

lim

x

x 3

2

x

1

lim x

x

3

1

x

.

z

2

x

lim 1

x

x

3

x

lim 1

x x

,

ln a.

e2

e

3

e5 .

7. Некоторые следствия замечательных пределов.

Непрерывность элементарных функций используется при вычисления

пределов. Ниже рассматриваются пределы, находящие применение в

дальнейшем.

1. Следствия первого замечательного предела.

tgx arcsin x arctgx

1) lim 1; 2) lim 1; 3) lim 1.

x0 x x0 x x0

x

так как

Вычислим их

tgx sin x 1 sin x 1

lim lim

lim lim 1

x0 x x0 x cos x x0 x x0

cos x

limcos x cos(lim x) cos0 1

x0 x0

(вследствие непрерывности косинуса в точке

2) Функция y sin x строго монотонна и непрерывна на отрезке

48

х 0);

; ,

2 2

обратная ей функция arcsin

x y также строго монотонна и

непрерывна на отрезке [ 1;1]. Кроме того, так как sin0 0, то условия


x0, y0

равносильны. Применяя правило замены переменной при

вычисления предела, получим

arcsin x

y

arcsin x

y 1

lim x sin y lim 1.

x0 x

y0sin

y sin y

x 0 y

0

lim

y0

y

arctgx

1) lim 1 вычисляется аналогично 2). Эти примеры

x0

x

обосновывают эквивалентность бесконечно малых функций.

(при x 0 ).

2. Следствия второго замечательного предела.

x ~ tgx ~ arcsin x ~ arctgx

log

a(1 x)

1)lim log

a

e,

x0

x

x

a 1

2)lim ln a,

x0

x

(1 x) 1

3)lim .

x0

x

Вычислим их:

в частности

в частности

49

ln(1 x)

( ae)lim 1;

x0

x

x

e 1

( ae)lim 1;

x0

x

1 1

log (1 )

x

x

a

x

1) lim limlog

a(1 x) log

a

lim(1 x) log

a

e.

x0 x x0 x0

Здесь использованы непрерывность логарифмической и сложной

функций, второй замечательный предел

x0

1

x

lim(1 x) e.

2) Функция строго монотонна и непрерывна на всей числовой

прямой, поэтому обратная функция

также строго монотонна и

непрерывна для

x имеем

ya

x

1

y 1. Так как при 0

xlog (1 y)

a

y 0, то условия

x 0,

y 0 равносильны. Далее вследствие правила замены переменных находим:

x

a 1 y;

x

a 1 y 1

lim x log

a(1 y) lim ln a.

x0 x y0log a(1 y) logae

x 0 y

0

Аналогично, используя непрерывность степанной функции и правило

замены переменных, вычислим

(1 x) 1

y

(1 x) 1

y

3) lim ln(1 x) ln(1 y) lim

x0 x

x0 y0

x

x 0 y

0

y ln(1 x)

y ln(1 x)

lim lim lim .

x 0 y 0 ln(1 y) x

y0 ln(1 y)

x0

x

Здесь обоснована эквивалентность бесконечно малых


x

x

x ~ ln(1 x) ~ e 1; xlog e ~ log (1 x); xln a ~ a 1;

x ~ (1 x) 1 ( x 0).

a

a

8. Раскрытие неопределенности по правилу Лопиталя

8.1. Расскрытие неопределенности вида

Определение. Говорят, что при

определенных на окрестности точки

неопределенность вида

0

,

0

если

неопределенность – значит вычислить

a

x

0

0

a

функции

отношение

f( x)

и

gx ( )

lim f ( x) lim g( x) 0.

xa xa

f( x)

lim

xa

gx ( )

f( x)

gx ( )

(если он существует).

Аналогично вводятся понятия неопределенности вида

x a 0( a 0), x ( ).

неопределенности вида

0

,

0

0

,

0

двух

представляет

Раскрыть эту

при

Следующая теорема дает правило раскрытия

при x

a.

f( x)

gx ( )

Теорема 1 (правило Лопиталя). Пусть функции и определены

и дифференцируемы на некоторой проколотой окрестности точки а , т.е.

для Кроме того, для всех

Пусть далее

xU 0 ( a).

lim f ( x) lim g( x) 0.

xa xa

g( x) 0

-

xU 0 ( a).

Тогда, если существует (конечный или бесконечный) предел

f( x)

lim ,

xa

g( x)

то существует также предел

f( x)

lim ,

xa

gx ( )

и выполняется равенство

f ( x) f ( x)

lim lim .

x a g( x) x a g( x)

Примеры.

2 1 2 1

x sin

sin

0

x

1) lim x

x

lim

. Действительно,

x0 sin x 0 x0

sin x

50


2 1

sin

1 1

x

2xsin cos

lim

x lim x x

x0 x0

sin x

cos x

существует

тем существует

1

limcos ,

x0

x

а предел

не существует (вследствие того, что не

1

lim2xsin

x0

x

2 1

x sin

1

lim x x

lim xsin lim 01

0.

x0 sin x x0 x x0

sin x

существует и равен нулю). Между

2) Пусть 0 1, тогда

1

x ( x ) x 1x

lim lim lim lim т.е.

x0 0 0 0

1

ln(1 x) x (ln(1 x))

x 1

x

x

1

x

при

ln(1 x) o( x )

Пусть

x o(ln(1 x))

1, тогда

x 0, если 0 1.

x

ln(1 x)

x

1

1

x

1

lim lim 1

lim x (1 x) 0,

x0 x0 x0

при

3) Двухкратным применением правила Лопиталя вычисляется предел

x 0,если

1.

2 2 2

x cos x 2xcos x x sin x 2cos x 4xsin x x cos x

lim lim lim 2.

x0 cos x 1 x0 sin x x0

cos

x

Примеры.

1)

1

cos x sin x 1

lim lim .

x0 2

x x0

2x

2

2) Следующее предельное значение вычисляется двухкратным

применением правила Лопиталя:

x sin x 1cos x sin x 1

lim lim lim .

x 0 3 x 0

2

x 3x x 0

6x

6

3) Трехкратным применением правила Лопиталя вычисляется

предельное значение

4 3 2

x 4x 12x

24

lim lim lim lim 12.

x 0

2

x 2cos x 2 x 0 2x 2sin x x 0 2 2cos x x 0 2sin x

Пример. Найти предел:

2

x 1ln

x

lim .

x1

x

e e

Решение:Числитель и знаменатель стремятся к нулю при x 1, а

потому имеем неопределенность вида 0/0. Воспользуемся правилом

Лопиталя, т. е. рассмотрим предел отношения производных заданных

функций:

т.е.

51


2

x 1

ln x 2x 1/ x 3

lim lim .

x1 x

x 1

x

e e e e

x

sin x

Пример. Найти предел: lim .

x0

3

x

Решение: Это - неопределенность вида 0/0. Имеем

x sin x 1cos x sin x 1

lim lim lim ,

x 0 3 x 0

2

x 3x x 0

6x

6

так как

sin x

lim 1.

x0

x

Пример. Найти предел:

Здесь правило Лопиталя применено дважды.

x

2

lim

x0

5

2x

x

Решение: Имеем долю полиномов без свободного члена, что в

предельной точке дает особенность вида 0/0. Для ее раскрытие по правилу

Лопиталя дифференцируем каждый полином пока не получим дробь, предел

которой можно вычислить подстановкой

2

2

2 x 2x

x x 2x

2 2

lim lim lim ;

x 0 5x

x 0 5x

x

0 5 5

8.2. Неопределенности вида

Определение. Говорят, что при

определенных на окрестности точки

неопределенности виду

, если

a

52

x

a

функции

отношение

f( x)

и

lim f ( x) lim g( x) .

xa xa

gx ( )

Аналогично вводятся понятия неопределенности вида

x a o ( a 0), x ( ).

f( x)

gx ( )

двух

представляют

,при

Для раскрытия этой неопределенности справедливо утверждение,

аналогичное теореме, доказанной в предыдущем пункте.

Теорема 2 (второе правило Лопиталя). Пусть функции и

определены и дифференцируемы на некоторой проколотой

точки , т.е. для x U 0

( a).

Кроме того, g( x) 0 для всех x

U 0

( a).

Пусть далее

lim f ( x) lim g( x) .

xa xa

Тогда, если существует (конечный или бесконечный) предел

то существует также предел

f( x)

lim

xa

gx ( )

f( x)

gx ( )

- окрестности

f( x)

lim ,

xa

g( x)


и выполняется равенство

Примеры.

x sin x ( x sin x)

1) lim lim .

x

x cos x x

( x cos x)

Действительно,

f ( x) f ( x)

lim lim .

x a g( x) x a g( x)

( x sin x) 1cos

x

lim lim

x

( x cos x) x

1sin

x

53

не существует. (В силу

того, что не существуют пределы limcos ,limsin ). Между тем, существует

предел

x

x

x

sin x sin x

1

1lim

xsin x 1 0

lim lim x x

x

1.

x

xcos x x

cos x cos x

1

1lim

10

x

x

x

2) Покажем, что при x любая степенная функция

растет быстрее логарифмической функции

Действительно, по правилу Лопиталя:

1

x x

lim lim lim x ,

x ln x x 1

x

x

ln x.

x

x ( 0)

т.е. при x функция является бесконечно большой

более высокого порядка, чем бесконечно большая функция ln x .

x

3) Покажем, что при x экспоненциальная функция e растет

быстрее, чем любая степенная функция Если n-

натуральное

число, то

x ( 0)

x ( 0).

n- кратным применением правила Лопиталя получим

x x x x

e e e e

lim lim lim ... lim .

x nx n( n 1) x n!

x n

x n1 x n2

x

Случай, когда -

натуральное число, сводится к рассмотренному, сели

учесть что

( здесь [ ] - целая часть числа ). Следовательно, при

-бесконечно большая более высокого порядка, чем

бесконечно большая функция x ,( 0).

Примеры.

x

x

x функция

1)

[ ] 1

e

x

1

ln x

lim x ln x lim lim x 2 lim x 0.

1

x

x

2

x00 x00 1

x 0 0

3

x00

2

2

2) n -кратным применением правила Лопиталя вычисляется предельное

значение

n n1 n2

x nx n( n 1) x n!

lim lim lim lim 0.

x x

x

x

x

x

x

x

e e e e


n

x

Пример. Найти lim , если n целое положительное число.

x

x

e

Решение: Это—неопределенность вида Применим правило

Лопиталя n раз:

54

/ .

n n1 n2

x nx n( n 1) x n( n 1)( n 2)...1

lim lim lim ... lim 0.

x x

x

x

x

x

x

x

e e e e

x/2

xe

Пример. Найти предел: lim .

x

x

x

e

Решение: В данном случае также имеет место неопределенность вида

/ .

Находим

x/2 x 1 x/2

x

x/2

e 1 e 2

xe

2 2 2

lim lim

lim

x x

x

x x

x e 1e x

e

x

1

2

2 1 1/ 2

lim lim 0.

x/2 x/2

2 x

e 2 x

(1/ 2) e

8.3. Неопределенности других видов:

Кроме неопределенностей видов

0 0

(0 ),( ),(1 ),(0 ),( ).

0

0

и

,

изученных выше,

встречаются еще и следующие:

Все эти неопределенности путем алгебраических преобразований

сводят к виду

0

0

или

.

0 0

(0 ),( ),(1 ),(0 ),( ).

первым неопределенностям. Каждая из них имеет вид

y

g( x)

[ f ( x)] ,

Покажем это, например, по отношению к трем

где при x a функция f( x ) стремится соответственно к 1, 0 или , а

функция стремится к или 0. Считая, что прологарифмируем

gx ( )

f( x) 0,

g( x)

выражение y [ f ( x)] , тогда ln y g( x)ln f ( x).

При x a выражение ln y g( x)ln f ( x)

представляет

неопределенность вида Эту неопределенность надо привести к виду

0

0

или

.

(0 ).

Вычислив предел выражения ln y g( x)ln f ( x)

, легко

g( x)

записать предел выражения y [ f ( x)] .

Заметим, что в любом из трех рассматриваемых случаев выражение

ln y g( x)ln f ( x)

представляет собой при х неопределенность вида 0

к неопределенности вида 0

или . Покажем, как это делается.

0

Итак, пусть z ( x) ( x),


Причем

lim ( x) 0,lim ( x) .

xa xa

вида

0 .

Перепишем

z ( x) ( x)

в виде

( x)

z ( x) ( x) .

1

( x)

Очевидно, выражение представляет собой при

0

Наша цель достигнута.

Примеры. 1) Вычислить

x

lim x .

x

0 0

x

a

Обозначим

ln x

ln y xln x . Применяя правило Лопиталя, будем иметь

1

x

Тогда

1

ln x

lim (ln y) lim lim x lim x

0.

1 1

2

x x

x00 x00 x00 x00

Отсюда ясно, что

2)

x0

1

2 e

x 1 x

lim(1 x ) .

lim y 1.

x

0 0

1

e 1x

Пусть

2

ln y ln y(1 x ).

x

1

2 e x

1

x

y(1 x ) .

Пользуясь правилом Лопиталя, получим

2x

2

ln(1 x ) 2

1

2x

limln y lim lim x lim

x0 x0 x

0

x

0

x

2

e 1 x x e 1 x

( e 1)(1 x )

2

lim 2.

x0

x 2 x

e (1 x ) ( e 1)2

x

Обозначим

Отсюда ясно, что

Пример. Вычислить

1

1

,

y x x тогда

2

lim y

e .

x0

1

1 x

x

x1

lim .

1

ln y ln x.

1 x

1

1 1

limln y lim ln x lim x lim 1.

x1 x1 1x

x1 1

x1

x

неопределенность

y

x

Это неопределенность вида (1 ).

1

ln

lim ln y

1

1 1 1

x

y x

Следовательно, lim x lim y lim e e e .

x1 x1 x1

e

x

.

Тогда

55


Пример. Найти предел:

2

x 5x4

lim

.

x

2

x 3x7

Решение: Делением числителя дроби на знаменатель выделим целую

часть:

2

x x x

2 2

x x x x

5 4 8 3

1 .

3 7 3 7

Таким образом, при

степень, основание которой стремится к единице, а показатель — к

бесконечности (неопределенность вида 1 ). Преобразуя функцию так, чтобы

использовать второй замечательный предел, получим

2

x

x x x

2

x

2

x x x x

5 4 8 3

lim lim1

x

3 7 3 7

x(8x3)

2

x 3x7 2

x 3x7

8x3

8x

3

lim 1

x

2

x 3x7

Так как

8x

3

0

2

x 3x7

x

2 3x7

8x3

8x

3

lim1

e.

x

2

x 3x7

x

x данная функция представляет собой

при

x

83/

x

x 3x713/ x7/

x

8x3

2 2

8 3

lim

x

1

.

x

2

x 3x7

x , то

8 3 / x

Приняв во внимание, что lim 8, получаем

x

2

13 / x7 / x

2

x

x 5x4

8

lim

e .

x

2

x

3x7

Пример. Найти предел:

/ ,

2

lim x ln x .

x0

Решение: Здесь мы имеем неопределенность вида 0 . Представим

произведение функций в виде частного, а затем, получив неопределенность

вида применим правило Лопиталя:

2 ln x 1/ x 1 2

limx ln x

lim lim lim x 0.

x 0 x 0

2

0

3

1/ x x 2 / x 2 x 0

Раскрытие неопределенностей

Пределы с неопределенностью также раскрываем по правилу

Лопиталя, но предварительно проводим определенные элементарные

действия над слагаемыми, чтобы перейти от разности бесконечно больших

функций к дроби.

Пример. Найти предел:

1 1

lim

x1

ln x x

1

56


Решение: Формулы ниже хорошо иллюстрируют как дважды

применяли дифференцирование числителя и знаменателя дроби, чтобы

избавиться от неопределенности 0 / 0.

1

1

1 1 1 ln 0 x 1 ln x

x x

lim lim lim lim x

x 1 ln x x 1 x 1 ( x 1)ln x 0 x 1

1

1

( x 1)ln x x

ln x ( x1)

x

x 1 0

x

1

1 1 1

lim lim lim .

x 1 xln x ( x 1) 0 x 1 x

xln x ( x 1)

1 ln x 11 0 11 2

1 1

Пример. Найти предел: lim

x0

sin x x

Решение: Имеем неопределенность вида , которую раскрываем

путем сведения дробей к общему знаменателю. Далее по правилу Лопиталя

вычисляем производные числителя и знаменателя, и так дважды.

1 1 x

sin x 0

x

sin x

lim lim lim

x0 sin x x x0 x sin x 0 x0

xsin

x

1

cos x 0

1

cos x

sin x

lim lim lim 0

x 0 sin x xcos x 0 x 0 sin x xcos

x

x

0 cos x cos x sin x

1 1 1

lim .

x1

ln x 11 0 11 2

И только когда избавляемся неопределенности выполняем подстановку

аргумента в предел.

1 1

Пример. Найти предел: lim .

x0

x

x e 1

Решение: Это — неопределенность вида . Для того чтобы найти

предел функции, приведем дроби к общему знаменателю, а затем, получив

неопределенность вида 0/0, применим правило Лопиталя:

x

x

1 1 e 1 e 1

lim

lim lim .

0

x

x 1 x 0

x x

1 x 0

x

x e e xe

e (2 x) 2

И только когда избавляемся неопределенности выполняем подстановку

аргумента в предел.

Пример. Найти предел:

x

lim(sin x) .

x0

0

Решение: Это —неопределенность вида 0. Обозначим данную

x

функцию через y , т. е. y (sin x) , и прологарифмируем ее:

lnsin x

ln y x lnsin x .

1/ x

Вычислим предел логарифма данной функции, применяя правило

Лопиталя (здесь имеем неопределенность вида /

):

57


2

lnsin x cos x / sin x x cos x

limln y lim lim lim

x 0 x 0 0

2

1/ x x 1/ x x 0 sin x

x

limx cos x

0.

x0

sin x

Следовательно,

0

lim ye

1.

x0

Пример. Найти предел:

2cos x

lim ( tgx) .

x

/2

Решение: Это - неопределенность вида

прологарифмируем:

2lntgx

ln y 2cos x ln tgx .

1/ cos x

Применяя правило Лопиталя, получим

2

lntgx

sec x / tgx

lim ln y 2 lim 2 lim

sec x sec x tgx

x /2 x /2 x

/2

sec x sec x tgx

2 lim 2 lim lim cos x 0,

x /2

2 x /2

2

tg x tgx sec

x x

/2

т. e.

0

lim ye

1.

x

/2

Пример. Найти предел:

ln x

lim(1 x) .

x0

0 . Положим

( tgx)

2cos x

Решение: Это - неопределенность вида 1. Логарифмируя и применяя

правило Лопиталя, получим

ln(1 x)

x

limln y limln x ln(1 x) lim

x0 x0 x0

1/ ln

2 2

1/ (1 x) xln x ln x

lim lim lim

x0 2

1/ ( xln x) x0 x 1 x0

11/

x

(2ln x) / x ln x 1/ x

lim 2lim 2lim 0.

x 0

2

0 0

2

1/ x x 1/ x x 1/

x

Таким образом,

lim y

0

e 1.

x0

[24, 25].

y

и

9. Разные задачи на вычисление пределов

Раскрытие неопределённостей видов "ноль делить на ноль" и

"бесконечность делить на бесконечность"

Пример Вычислить предел отношения двух функций, пользуясь

правилом Лопиталя:

2

x 3x2

lim .

x2

2

ln x 3

Решение: Подстановка в заданную функцию значения x=2 приводит к

неопределённости вида 0/0. Поэтому производную каждой функции и

получаем

58


2

3 2 0 x 2 3x 2

x x 2x

3 2x 3 x

2 3 1

lim lim lim lim .

x 2 2

ln

2 2 2

x

3

0 x x 2

2x

x

2x

4

ln

x 3

2

x 3

В числителе вычисляли производную многочлена, а в знаменателе -

производную сложной логарифмической функции. Перед последним знаком

равенства вычисляли обычный предел, подставляя вместо икса двойку.

Пример .Вычислить предел отношения двух функций, пользуясь

правилом Лопиталя:

sin x

lim .

x0

sin 2x

Решение: Подстановка в заданную функцию значения x=0 приводит к

неопределённости вида 0/0. Поэтому вычисляем производные функций в

числителе и знаменателе и получаем:

sin x 0 sin x

cos x 1

lim lim lim .

x0 sin 2x

0 x0 x0

sin 2x

2cos2x

2

Пример . Вычислить предел отношения двух функций, пользуясь

правилом Лопиталя:

x

e 1

lim .

x0

sin3x

Решение: Подстановка в заданную функцию значения x=0 приводит к

неопределённости вида 0/0. Поэтому вычисляем производные функций в

числителе и знаменателе и получаем:

x

x

1 0 e 1

x

e e 1

lim lim lim .

x 0 sin3x

0 x 0 x

sin3x

0 3cos3x

3

Пример. Найти предел:

ln x

lim .

x

x

[26, 27].

Решение: Подстановка в заданную функцию значения икса, равного

плюс бесконечности, приводит к неопределённости вида ∞/∞. Поэтому

применим правило Лопиталя:

ln x

ln x

1

lim lim lim 0.

x x x x x

x 1

Замечание. Переходим к примерам, в которых правило Лопиталя

приходится применять дважды, то есть приходить к пределу отношений

вторых производных, так как предел отношения первых производных

представляет собой неопределённость вида 0/0 или ∞/∞.

Пример .Вычислить предел отношения двух функций, пользуясь

правилом Лопиталя:

2

x 2x

2 1

lim lim lim lim lim 0.

x

2x

x

2x

2x 2x

e 2e x x 4e x

2e

59


Здесь правило Лопиталя применено дважды, поскольку и предел

отношения функций, и предел отношения производных дают

неопределённость вида ∞/∞.

3 2

3x

12x

Пример. Найти предел: lim .

x0

3 2

7x

x

Решение. Находим

3 2 2

3x 12x 0 9x 24x 0 18x

24

lim lim lim 12.

0

3 2 x 0

2

7x x 0 x 21x 2x 0 x 0

42x

2

Здесь правило Лопиталя применено дважды, поскольку и предел

отношения функций, и предел отношения производных дают

неопределённость вида 0/0.

Пример. Найти предел:

Решение: Находим

ln x

lim .

x0

ctgx

2

ln x 1sin x 0 2sin x cos

x

lim lim lim 0.

x 0 ctgx

x 0 x 1 0 x

0

1

Здесь правило Лопиталя применено дважды, поскольку и предел

отношения функций, и предел отношения производных сначала дают

неопределённость вида - ∞/∞, а затем неопределённость вида 0/0.

Пример. Найти предел:

Решение: Находим

x

2

lim .

x

2

x

x x x 2

2 2 ln 2 2 ln 2

lim lim lim .

x 2x

2

x 2

x x

Здесь правило Лопиталя применено дважды, поскольку и предел

отношения функций, и предел отношения производных сначала дают

неопределённость вида ∞/∞, а затем неопределённость вида 0/0.

Раскрытие неопределённостей вида «ноль умножить на бесконечность»

Пример. Найти предел:

Решение. Находим

lim

x0

x

2

ln x

2

2

ln x

x

limx

ln x 0

lim lim 0

x0 x0 1

x0

2

2

x

Здесь неопределённость вида 0 мы преобразовали к виду

1

limln x , lim ,

x0 x0

2 а затем применили правила Лопиталя.

x

Пример. Найти предел: lim x

x

tg

x 2

так как

60


Решение. Находим

x x 0

1

2 x

lim

xtg

0 lim lim lim2sin 2

x 2 x x

0 x 1 1 x

ctg

2

2 2 x

sin

2

2

В этом примере использовано тригонометрическое тождество

tgctg

1

Раскрытие неопределённостей видов "ноль в степени ноль",

"бесконечность в степени ноль" и "один в степени бесконечность"

Неопределённости вида или 1 обычно приводятся к виду 0/0

0,

0 0

или ∞/∞ с помощью логарифмирования функции вида

g x

f x

Чтобы вычислить предел выражения f x , следует использовать

логарифмическое тождество частным случаем которого является

ln b

a

b

a

log

a b

b,

и свойство логарифма loglog .

Используя логарифмическое тождество и свойство непрерывности

функции (для перехода за знак предела), предел следует вычислять

следующим образом:

k

ab k

ab

lim ( )

g ( x) ln f ( x) ( ) lim f x e ( ) ln ( )

0

g x f x x x

lim e e

.

xx0 xx0 xx0

lim g ( x) ln f ( x)

Отдельно следует находить предел выражения в показателе степени и

возводить e в найденную степень.

Пример. Вычислить, пользуясь правилом Лопиталя:

Решение. Получаем

1

1

ln(1 ) lim

ln(1 x ) x

x x

x 0

x e e e x

x

lim 1 1 lim lim

x0 x0 x0

1 ln(1 x)

Вычисляем предел выражения в показателе степени

1

ln(1 x) 0 1 1

lim lim x

lim 1

x0 x 0 x0 x x0

1

x

x0

Итак,

1 x 1

lim 1 x e e.

Пример . Вычислить, пользуясь правилом Лопиталя :

Решение: Получаем

lim sin xln

x

sin x

sin x 0 ln x sin xln

x

x 0

x e e e

lim 0 lim lim

x0 x0 x0

Вычисляем предел выражения в показателе степени

x0

.

g x

1

lim 1

x

lim x

x0

sin x

x

61


1

2

ln x 0

sin x

limsin x ln x 0

lim lim x lim

x 0 x 0 1

0 x0 cos x x0

x

cos x

2

sin x

sin x

x x x x

lim lim lim lim 0

2 2

sin 1 sin 0 2sin cos

x x0cos x x0

x 0

1

x0

x0

Итак

sin x 0

lim x e

1

x0

lim ln x

Пример .Вычислить, пользуясь правилом Лопиталя : 1

x

Решение: Получаем

1 1 1

ln ln x

x

ln ln lim

x x x

lnln

x

0

x

x

lim ln x lim e lim e e

x x x

Вычисляем предел выражения в показателе степени

1 1

lnln x

ln

1

lim lim x x

lim 0

x x x x x

x ln

x

Итак

1 0

lim ln x x e

1

x

x

Раскрытие неопределённостей вида "бесконечность минус

бесконечность"

Это случаи, когда вычисление предела разности функций приводит к

неопределённости "бесконечность минус бесконечность":

lim( f ( x) g( x)) ( )

xx

0

Вычисление такого предела по правилу Лопиталя в общем виде

выглядит следующим образом:

1 1

g( x) f ( x)

lim( f ( x) g( x)) ( ) lim

xx0 xx0

1 1

f ( x) g( x)

1 1

1 1

0 g( x) f ( x)

lim lim

x x0 1 1

0 xx0

1 1

f ( x) g( x)

f ( x) g( x)

В результате таких преобразований часто получаются сложные

выражения, поэтому целесообразно использовать такие преобразования

разности функций, как приведение к общему знаменателю, умножение и

62


деление на одно и то же число, использование тригонометрических тождеств

и т.д. [28, 29].

Пример. Вычислить, пользуясь правилом Лопиталя:

63

1 cos3x

lim .

x0

2 2

x

x

Решение: Пользуясь вышеперечисленными рекомендациями, получаем

1 cos3x 1

cos3x

0

lim ( ) lim

x0 2 2

x 0

2

x x

x 0

3sin3x

0 9cos3x

9

lim lim .

x0 2x

0 x0

2 2

Пример . Вычислить, пользуясь правилом Лопиталя:

lim 2 1 2

4

x

lim

x

x x x

Решение: Пользуясь вышеперечисленными рекомендациями, получаем

lim 2 1 2

4

x

x x x

2 2 2 2

x 1 x 4x x 1 x 4x

2 2

x 1 x 4x

2 2

x 1 x 4x 4x

1 4

lim lim 2.

x

2

2 2 x

2 2

x 1 x 4x x 1 x 4x

Метод логарифмирования раскрытия неопределенностей в пределах

Пример . Вычислить предел по формуле Лопиталя:

lim x

x

1/(1 x)

Решение: Предел имеет особенность типа Для применения метода

1/(1

логарифмирования за новую функцию обозначим y x x)

. Делее

логарифмируем обе части, получим

1/(1 x)

ln( y) ln( x ) ln( x) / (1 x).

По правилу Лопиталя раскрываем неопределенность вида 0 / 0

1

ln( x) 0

1

limln y lim lim x lim 1

x x1 1x

0

x1 1 x1

x

Это еще не конечный ответ, чтобы найти y нужно экспоненту

поднести к степени равному найденному пределу.

ln( y) 1 y e .

e

Пример. Раскрыть неопределенность по правилу Лопиталя:

2

lim(arcsin x) x

x

0

1 1

0

Решение: Имеем неопределенность типа ноль в степени ноль 0.

Поступаем по схеме для показательных функций, а именно – логарифмуемое

выражение в лимите.

1.


y

2

(arcsin( x)) ;

ln( y) 2x ln(arcsin( x)).

Для раскрытия неопределенности используем правило Лопиталя

дважды

ln(arcsin( x))

ln(arcsin( x))

lim 2xln(arcsin( x)) 2 lim 2 lim

1

1

x

x

x0 x0 x0

1 1 1

1

2

arcsin( )

2

1

arcsin( ) 0

x

x

2 lim

x

x

2 lim 2 lim

x0 1 x0 1 0

x0

(arcsin( x))

2 2

x

x

2x

1

2 lim 4 lim x 1

2

x 4 0 1 0

x0

x0

2

1

x

Здесь нужно найти производную от функции

сложной функции, помните об этом.

Пример. Найти предел, пользуясь правилом Лопиталя:

64

ln(arcsin( x)),

tan x

limsin x .

x

2

1.

как от

Решение: Подставки аргумента равного 90 градусам дает

неопределенность вида единицы в степени бесконечность Согласно

алгоритму вычисления пределов, функцию под лимитом следует

прологарифмировать. Далее найти предел логарифма, а потом экспонента в

степени полученного значения и будет ответом к заданию. Проведем

вычисления

y

sin( x)

tan( x) ;

( )

ln( y) ln(sin( x)) tg x tan( x) ln(sin( x)).

Воспользуемся правилом Лопиталя, предварительно заменив тангенс

котангенсом по формуле

1

tg( x) .

cot( x)

lnsin( x)

lnsin( x)

lim tan( x)lnsin( x) lim lim

cos( x)

cos( x)

x x x

2 2 2

1

cos( x)

sin( x)

lim limsin( x)cos( x) 0 1

0.

x

1

x

2

2

sin ( x)

Но это еще не ответ, нужно потенцировать 0.

0

ln( y) 0 y e e.

Сведение неопределенностей в пределах под правило Лопиталя.


Пример. Мнимая подстановка x=2 в функцию дает неопределенность

вида 0/0, поэтому применяем правило Лопиталя однократно, а далее без

труда вычисляем лимит.

1

7 x

3 0 7x

3

lim lim lim

2 7

x

x 2 x 2 0 x 2 x

x 2

2 1

1 1 1 1

lim

x2

2 7 x 2 7 x

23 6

Пример. Найти предел :

x

4

t gx ( ) 1

lim sin(4 x )

Решение: Подстановкой убеждаемся, что имеем неопределенность вида

0/0. По формуле Лопиталя вычисляем производную числителя и знаменателя

по переменной, чтобы раскрыть неопределенность.

1

2

tan( x) 1 0

cos ( x)

1 1 1

lim lim lim

2

2

x sin(4 x)

0

x 4cos(4 x)

x

4cos(4 x)cos ( x) 2

4 4 4

2

Пример. Найти предел:

2

x

lim 2

x

x

4 ( 1)

2

Решение: В числителе имеем , в знаменателе 2.

Поскольку аргумент стремится к бесконечности, то прямая

подстановка дает особенность вида

x

2

.

производные числителя и знаменателя по " ".

x

x

Ее раскрываем дважды беря

2

2

x

x 2x

2 x

lim lim lim lim

x x

x

2

x x

x 2

2 ln 2 ln 2 x

x

2

2 1 2

lim 0 0

2

ln2 x

x

2 ln2 ln 2

Пример. Найти предел:

[30, 31].

lim

x

e

x

x

a

Решение: Имеем дробь с функций В такого сорта примерах

правило Лопиталя применяют до тех пор, пока в знаменателе не получим

факториал числа a !

x

x

x

e

x

e

e

e

lim = lim lim lim

x a

x 0

x

a 1

x

a

ax x

a1

x

a x

x a

e , x .

65


x

x

e

e

lim

lim

x

a2

a ( a 1) x x

a!

Ограничений на количество повторных применений правила Лопиталя

нет, находим производные до тех пор, пока имеем одну из

неопределенностей 0 0 или .

Пример .

a

x

lim

x

ln x

Решение: Очередное задание на раскрытие неопределенности вида

решаем путем дифференцирования отдельно числителя и

знаменателя ln( ).

x

ln x

x

ax

1

x

a

a1

lim lim a1

a lim x x

x x x

Пример . Найти предел:

arct g(7 x)

lim

x0

3x

e 1

3x

Решение: Имеем долю функций f ( x) arct g(7 x) и g( x) e 1.

В нуле

они дают неопределенность вида 0/0, поэтому имеем все основания

применить правило Лопиталя.

1

7

arct g(7 x) 0

2

1 49 7

lim = lim x .

x0 3x

0

3x

e 1 0 x

3e

3

Поскольку выражения

равен 7 3.

Пример. Найти предел:

e

1 и 1 (1 49 x ) 1

3x

2

2

ln x

lim

x

3

x

когда

x

x 0, то предел

Решение: Согласно алгоритму, чтобы раскрыть неопределенность ∞/∞

дважды применяем правило Лопиталя.

1 2

2

2ln x

ln x

2ln

x

2

lim lim x lim lim x lim 0

x 3

x

2 2 3

x

3x x 3

x 3

3 3x x

x

9x

Пример. Найти предел:

x0

lim xln( x)

Решение: Переходим от неопределенности вида ноль умножить на

бесконечность к неопределенности бесконечность разделить на

бесконечность, которую раскрываем по правилу Лопиталя через

дифференцирование числителя и знаменателя дроби. Внимательно

посмотрите схему перехода от одной неопределенности к другой и

запомните, что когда имеем произведение логарифма на другую функцию, то

в знаменатель переносим последнюю, а не логарифм.

a

66


ln( x)

ln( x)

lim xln( x) 0 lim lim

x0 x0 1

x0

1

x

x

1

x x x

1 1

32

2 x

lim 2lim 1 ( 3/2)

2lim 2 0 2 0 0.

x0 x0 x0

Пример. Найти предел:

x

lim( e 1) ctg( x)

x0

Решение: Прямая подстановка дает неопределенность ноль умножить

на бесконечность 0 .

Чтобы свести пример к применению правила Лопиталя в

искусственный способ котангенс переносим в знаменатель дроби, а далее

заменяем

1

( ).

ctg( x)

tg x

67

Таким образом получаем особенность в виде доли

бесконечно малых функций, раскрываем дифференцированием по Лопиталю

и подстановкой

x 0.

x x x

x

e 1 e 1 e 1 0

lim( e 1) ctg( x) lim lim lim

x0 x0 1 x0 1 x0

tg( x)

0

ctg( x) ctg( x)

x

e 1

x

0

e e 1

lim lim 1

x0 x0

1 1 1

tg( x)

2 2

cos ( x) cos (0) 1

Правило Лопиталя для случая предела двух бесконечно малых величин.

Пусть функции

имеют производные (то есть дифференцируемы)

в некоторой окрестности точки a. А в самой точке a они могут и не иметь

производных. При этом в окрестности точки a производная функции g(x) не

равна нулю

и пределы этих функций при стремлении x к

значению функции в точке a равны между собой и равны нулю:

f ( x) и g( x)

( g( x) 0)

lim f ( x) lim g( x) 0

xa xa

Тогда предел отношения этих функций равен пределу отношения их

производных:

f ( x) f ( x)

lim lim 0.

x a g( x) x a g( x)

Правило Лопиталя для случая предела двух бесконечно малых величин.

Пусть функции имеют производные (то есть дифференцируемы) в некоторой

окрестности точки a. А в самой точке a они могут и не иметь производных.

При этом в окрестности точки a производная функции g(x) не равна нулю и

пределы этих функций при стремлении к значению функции в точке a равны

между собой и равны бесконечности:


lim f ( x) lim g( x) .

xa xa

Тогда предел отношения этих функций равен пределу отношения их

производных:

f ( x) f ( x)

lim lim 0.

x a g( x) x a g( x)

68

/

Иными словами, для неопределённостей вида 0 / 0 или предел

отношения двух функций равен пределу отношения их производных, если

последний существует (конечный, то есть равный определённому числу, или

бесконечный, то есть равный бесконечности).

Замечания.

x

a.

0 / 0 или

f ( x) и g( x)

1. Правила Лопиталя применимы и тогда, когда функции

не определены при

2. Если при вычислении предела отношения производных функций

f ( x) и g( x)

снова приходим к неопределённости вида

, то

правила Лопиталя следует применять многократно (минимум дважды).

3. Правила Лопиталя применимы и тогда, когда аргумент функций

(икс) стремится не к конечному числу a, а к бесконечности .

0 / 0 и

/

x

/

К неопределённостям видов

могут быть сведены и

неопределённости других видов.

Одним из самых мощных методов раскрытия неопределенностей и

вычисления пределов функций является использование правила Лопиталя.

Оно позволяет раскрывать неопределенности вида 0 / 0 или /

или бесконечно удаленной точке, которую мы обозначим как . Правило

Лопиталя заключается в том, что мы находим производные числителя и

знаменателя дроби. Если существует предел, то существует равный ему

предел.

Если после дифференцирования мы опять получаем неопределенность,

то процесс можно повторить, то есть применить правило Лопиталя уже к

пределу. И так далее, до раскрытия неопределенности.

Для применения этого правила, должна существовать такая проколотая

окрестность точки , на которой функции в числителе и знаменателе

являются дифференцируемыми и функция в знаменателе и ее производная не

обращается в нуль.

Применение правила Лопиталя состоит из следующих шагов.

1) Приводим неопределенность к виду 0 / 0 или /

. Для этого, если

требуется, выполняем преобразования и делаем замену переменной. В

результате получаем предел вида .

2) Убеждаемся, что существует такая проколотая окрестность точки x

0

,

на которой функции в числителе и знаменателе являются

дифференцируемыми и знаменатель и его производная не обращаются в

нуль.

3) Находим производные числителя и знаменателя.

x 0

в конечной

x 0


4) Если имеется конечный или бесконечный предел , то задача решена.

5) Если предела не существует, то это не означает, что не существует

исходного предела. Это означает, что данную задачу решить с помощью

правила Лопиталя нельзя. Нужно применить другой метод.

6) Если в пределе вновь возникает неопределенность, то к нему также

можно применить правило Лопиталя, начиная с пункта 2).

Как указывалось выше, применение правила Лопиталя может привести

к функции, предела которой не существует. Однако это не означает, что не

существует исходного предела [32, 33].

69


2.3 Педагогический эксперимент

Одним из этапов научного исследования является педагогический

эксперимент. В данном параграфе описывается методика проведения

эксперимента и проводится анализ результатов экспериментальной работы,

целью которой была проверка выдвинутой гипотезы.

Педагогический эксперимент проводился в течение 2020 -2022 гг. и

содержал три этапа: констатирующий, поисковый и обучающий.

Первый этап - констатирующий эксперимент был начат в 2020 году. На

этом этапе изучалась и анализировалась психолого-педагогическая и научнометодическая

литература по проблеме исследования, формулировалась

гипотеза, цель и задачи исследования.

Основными задачами этого этапа эксперимента являлись:

- изучение и обобщение опыта преподавания элективных курсов по

математике в школе;

- анализ школьных программ по математике и педвузовских программ

по математическому анализу с целью выявления связи основных понятий

теории пределов со школьным курсом математики;

- обоснование необходимости изучения элементов высшей алгебры на

факулвьтативных занятиях в средней школе;

Для исследования проблемы были использованы разнообразные

методы исследования: анализ психолого-педаголической, математической и

методической литературы, школьных программ по математике и

педвузовских программ по алгебре; анкетирование студентов, учителей

школ, беседы с ними; изучение и обобщение педагогического опыта;

теоретическое обобщение результатов исследования.

В ходе проведения эксперимента учителям общеобразовательных

школ, студентам была предложена анкета (см. приложение 6). С ее помощью

представилась возможность более обстоятельно выяснить мнения и

пожелания учителей по постановке элективного курса.

В результате анкетирования было установлено, что организация

элективных курсов в школе осуществляется. Большинство учителей отдают

предпочтение тем темам, которые тесно связаны с основным курсом

математики. Выяснилось, что при подготовке и проведении занятий

элективного курса у учителей возникают трудности, связанные с отсутствием

соответствующей учебной литературы, а также с методикой проведения

занятий.

В результате проведенной на этом этапе работы было установлено, что

уровень математической подготовки выпускников школ в последнее время

стал снижаться. Этот вывод был сделан на основе обобщения

педагогического опыта, теоретических; исследований, а также в результате

наблюдений за учебным процессом и проведения анкетирования студентов.

Студентам первого курса в самом начале обучения была предложена

следующая анкета:

70


1. Что такое последовательность? Перечислите известные вам

числовые последовательности.

2. Предел числовой последовательности?

3. Предел функции?

4. Некоторые методы и приемы вычисления пределов функции.

Анализ ответов на вопросы анкеты свидетельствуют о наличии

затруднений при ответах на вопросы 2 и 4, требующие привлечения

обобщенных знаний, умения подметить аналогию,

В целом в знаниях выпускников наблюдалось отсутствие

структурности, т.е. отсутствие взаимосвязи, как между отдельными

понятиями, так и разделами школьного курса математики.

Результаты ответов представим в таблице:

Таким образом данные анкетирования свидетельствуют о том, что у

выпускников средних школ обнаружились недостатки в умении

системапизироватъ изученный материал, абстрагировать и обобщать его.

Кроме того, беседы с учащимися школ показали, что у них нет

представления о пределе функции и методах его вычисления.

Полученные в ходе эксперимента результаты позволили

сформулировать гипотезу исследования: школьный курс математикой

располагает большими возможностями для изучения элементов теории

предела на занятиях элективного курса в старших классах. Факультативный

курс «Некоторые методы и приемы вычисления пределов функции» будет

способствовать повышению прочности знаний учащихся по предмету,

уровня интеллектуального развития школьников. Изучение приемов

вычисления пределов функции подготовит учащихся к пониманию

современных научных идей, тем самым приблизит школьное преподавание к

школьной науке и ее приложениям.

Намеченная нами проблема и выдвинутая гипотеза помогли выбрать и

наметить методику дальнейшего исследования, направленную на

установление истинности гипотезы.

На втором, поисковом, этапе эксперимента (2021-2022 гг.) решались

следующие задачи:

1. Разработать программу и содержание элективного курса.

2. Отобрать и составить комплекс задач для осуществления учебной

деятельности.

3. Разработать методику изучения тем элективного курса.

В ходе проведения эксперимента необходимо было:

71


1. Уточнить содержание, объем и структуру построения учебного

материала.

2. Выяснить, в чем состоят особенности методических приемов при

изучении элементов теории пределв по предложенной системе.

3. Проверить эффективность разработанной нами методики изучения

элементов высшей алгебры на факультативных занятиях в средней школе.

Эффективность методики проверялась по следующим критериям:

доступность материала элективного курса качество его усвоения и

влияние данного курсa на интеллектуальное развитие учащихся. Для

оценки доступности содержания факультативного курса использовали

устный опрос учащихся, наличие активности при изучении материала на

занятиях.

Опытно-экспериментальное обучение показало, что содержание тем

«Предел функции», «Некоторые методы и приемы вычисления

пределов функции» затруднений в понимании не вызвали.

Наиболее трудными для учащихся оказались некоторые вопросы

абстрактной алгебры («Неопределенности разных видов»).

Поэтому необходима более тщательная предварительная подготовка их

введения, заключающаяся в хорошо продуманной системе подводящих

упражнений.

Для оценки качества усвоения содержания элективного курса

учащимся использовали устный опрос участников элективных курсов и

анализ результатов контрольных работ.

Среди качественных характеристик полноценных знаний выделим

следующие:

1. Полнота (количество всех знаний об изучаемом объекте).

2. Обобщенность. Обобщение знаний - процесс, в ходе которого

выделяются и объединяются общие существенные черты различных

объектов.

Именно в выделении главного, существенного в учебной информации

особенно ярко проявляет себя обобщение.

При ивучении данного элективного курса учащииеся знакомятся с

такими общими понятиями как группа, кольцо, поле, с помощью которых

мысленно обозреваются и изучаются с новой точки зрения, ранее усвоенные

знания, которые в рамках более узких понятий остались нераскрытыми.

Следовательно, элективный курс позволяет обобщить знания

учащихся:

Степень обобщенности знаний проверялась в ходе переноса их на

решение новых учебно-практических задач.

3. Прочность знаний. Прочность знаний учащихся означат длительное

сохранение в памяти изучаемых знаний, воспроизводимость IIX в

необходимых случаях.

Сведение всего изученного учащимися в единый комплекс, выделение

в нем главного и второстепенного, определение их взаимосвязи позволяет

72


систематизировать знания учащихся по математике. За счет систематизации

изученных ранее знаний и выделения наиболее важных мыслей на

факультативных занятиях можно добиться более успешного усвоения

программного материала, тем самым повысить прочность знаний учащихся.

Прочность усвоения учебного материала устанавливается по результатам так

называемых отсроченных проверок.

Анализ выполнения контрольных работ проводился следующим

образом: за каждое правильно выполненное задание учащийся получал 1

балл, если задание выполнено не совсем верно, т.е. допущены

незначительные опшбки, то начисляем 0,5 балла. Если в задании допущены

грубые опшбки или вообще к его выполнению не приступали, то - 0 баллов.

Определение оценки осуществлялось по таблице, которая была

составлена

исходя из формулы:

k==n/m, где п - полученное количество баллов,

m - максимальное число баллов,

к – коэффисия ент усвоения учебного материала.

Результаты контрольных работ, приведенных в таблице ниже,

свидетельствуют о доступности предлагаемого курса для учащихся.

Содержание элективного курса готовит учащихся к успешному

овладению материалом на последующих этапах обучения, поскольку имеет

место связь изучаемого в школе материала с материалом, который предстоит

изучать в вузе. Тем самым осуществляется преемственность обучения.

73


ЗАКЛЮЧЕНИЕ

Проведенное по теме теоретическое и экспериментальное исследование

позволило сделать следующие основные выводы:

1. Проведенный научно-методический анализ по проблеме исследования

показал возможность и целесообразность изучения элементов теории

пределов. Анализ содержания школьного курса показал, что в самой

программе по математике заложено предварительное знакомство со многими

понятиями теории пределов. Это послужило отправным пунктом при

решении вопроса о содержании элективного курса «Раскрытие

неопределенностей при вычислении пределов» для старшеклассников.

2. Элективный курс «Раскрытие неопределенностей при вычислении

пределов» содействует уменьшению разрыва между современнее и школьной

математикой посредством ввода основных понятий высшей математики. При

его изучении углубляются и расширяются знания учащихся по математике,

обобщается и приводится в систему ранее изученный материал, повышается

прочность знаний учащихся.

3. Изучение элементов теории пределов в школьном курсе способствует

общему развитию учащихся, повышают уровень их математического

образования и тем самым создает благоприятные условия для успешного

обучения в вузе.

4. Обучающий эксперимент показал, что элективный курс «Раскрытие

неопределенностей при вычислении пределов» доступен для учащихся и

может быть рекомендован для изучения в средней школе. Введение

спецкурса в образовательный процесс педагогического университета дает

студентам теоретическую основу для ведения элективного курса в школе по

предлагаемой тематике, подкрепленную практическими знаниями и

умениями. В процессе выполнения диссертационного исследования

получены следующие результаты:

1. Для обоснования возможности и целесообразности изучения элементов

теории пределов была установлена связь основных понятий предела с

основным курсом математики, проанализированы особенности психического

развития учащихся старших классов.

2. Составлена программа и разработано содержание элективного курса

«Раскрытие неопределенностей при вычислении пределов».

3. Разработаны методика и дидактический материал по изучению элементов

теории пределов в средних общеобразовательных учреждениях.

4. Разработаны программа и методические рекомендации к проведению

спецкурса «Раскрытие неопределенностей при вычислении пределов».

5. Экспериментально установлена эффективность разработанной системы по

изучению элементов теории пределов в органической связи с основным

школьным курсом математики.

74


СПИСОК ИСПОЛЬЗОВАННЫХ ИСТОЧНИКОВ

Ф.7.38-02

1 Закон Республики Казахстан О науке от 18 февраля 2011 года № 407-

IV

2 Письменный Д.Т.Конспект лекций по высшей математике: полный

курс – 9-е изд. М.: Айрис- пресс, 2009.- 608с.

3 Виленкин, Н.Я. Алгебра и математический анализ для 11 класса:

Учеб. пособие для учащихся шк. и классов с углубл. изуч. математики / Н.Я.

Виленкин и др.. М.: Мнемозина, 2000. - 287 с

4 Колмогоров А.Н. Алгебра и начала анализа: 10-11 кл.: учеб. для

общеобразоват. учреждений / А.Н. Колмогоров и др.; под ред. А.Н.

Колмогорова. 15-е изд.- М.: Просвещение, 2006. - 384 е.: ил.

5 Журбенко Л. Н. Математика в примерах и задачах : учебное пособие

для студ. вузов, обучающихся по техническим спец.; Допущено МОН РФ / Л.

Н. Журбенко [и др.]. - М. : ИНФРА-М, 2011. - 372 с. - (Высшее образование)

6 Демидович Б.П.Численные методы анализа. Приближение функций,

дифференциальные и интегральные уравнение : учебное пособие для студ.

вузов / Б. П. Демидович, И. А. Марон, Э. З. Шувалова ; под ред. В.П.

Демидовича. - СПб. : Изд-во "Лань", 2010. - 400 с. : ил. - (Учебники для

вузов. Специальная литература)

7 Яковлева Г.Н. Пособие по математике для поступающих в вузы, М.:

Наука,2011.- 607с.

8 Крупич В.И. Теоретические основы обучения решению школьных

математических задач:Дисс. ...док.пед.наук.–М.:1992.-380 с.

9 Ильин В.А. и др. Математический анализ. Начальный курс

/В.А.Ильин, В.А.Садовничий, В.Х.Сендов. Под.ред. А.Н.Тихонова. -М.: Издво

МГУ, 1985. - 662 с.

10 Абылкасымова А.Е., Кучер Т.П., Корчевский В.Е., Жумагулова З.А.,

Алгебра и начала анализа: Учебник для 10 класса ЕМН, Алматы: Мектеп,

2019; -275с.

11 Темиргалиев Н., Введение в математический анализ, Астана, 2015;-

388с.

12 Круглов Е.В., Мамаева Н.А., Таланова Е.А., Некоторые приемы

вычисления пределов Нижний Новгород, 2018;

13 Матвеева Т.А., Рыжкова Н.Г., Математический анализ,

Екатеринбург, 2017;

14 Самочернова Л.И., Высшая математика, Томск, 2005, -355с.

15 Альпин Т.Ю., Егоров А.И., Кашаргин П.Е., Сушков С.В.,

Практические занятия по математическому анализу, Казань, 2013.-365с.

16 Берман Г.Н. Сборник задач по курсу математического анализа.

Издание 22-е, переработанное.- СПб.: Изд-во «Профессия», 2001. – 432с.

17 Выгодский М.Я. Справочник по элементарной математике. Издание

17-е стереотипное. – М.: Наука, ГРФМЛ, 1966. – 424 с.

75


18 Данко П.Е., Попов А.Г., Кожевников Т.Я. Высшая математика в

упражнениях и задачах. Ч.1. Издание 4-е, испр. и доп. – М.: Высшая школа,

1986. – 304 с.

19 Кудрявцев Л.Д. Математический анализ. Т.1. – М.: Наука, ГРФМЛ,

1970. – 590 с.

20 Юлдашева С., Дуйсебаева П.С., Алибекова Ж.Д.Решение задач на

нахождение предела функции и его свойств. Международная научнопрактческая

конференция «Ауэзовские чтения–20: Посвященной 125-летию

классика казахской литературы М.О. Ауэзова»

21 Юлдашева С., Сабырханова П.Ш., Дуйсебаева П.С. Раскрытие

неопределенности с помощью правила лопиталя. Международная научнопрактческая

конференция «Ауэзовские чтения–20: Посвященной 125-летию

классика казахской литературы М.О. Ауэзова»

22 Зорич В. А. Математический анализ. Часть I. — 6-е изд, дополн.—

М.: МЦНМО, 2012. — XVIII + 702 с.

23 Данко П.Е., Папов А.Г., Кожевникова Т.Я. Высшая математика в

упражениях и задачах.В 2-х ч.Ч.1: Учеб. Пособие для втузов.- 5-е изд.- М.

Высш.шк.,1999.-304с.

24 Ильин В.А., Позняк Э.Г. Основы математического анализа. Ч.1.

Издательство «Наука», главная редакция физико-математической

литературы. Москва., 597с.

25 Кабдыкайыр К. Курс математики: Учебник,- Изд.4-е, доп, и

перераб.- Алматы:2005.-496с.

26 Шыныбеков А.Н., Шыныбеков Д.А., Жумабаев Р.Н. Алгебра и

начала анализа. Учебник для 10 класса общеобразовательной школы

естественно-математического направления. – Алматы: Атамұра, 2019 г. – 272

с.

27 Малахов А.Н. Высшая математика: Учебно-методический комплекс

/ А. Н. Малахов, Н. И. Максюков, В. А. Никишкин. - М.: Изд. центр ЕАОИ,

2009 г. - 396 с.

28 Фихтенгольц Г.М. Курс дифференциального и интегрального

исчисления: Том 1 / Г.М. Фихтенгольц. - М.: Книга по Требованию, 2013 г. -

608 с.

29 Белл Э. Т. ТВОРЦЫ МАТЕМАТИКИ: Предшественники современной

математики: Пособие для учителей. М.: «Просвещение», 1979 г. -

256 с.

30 Рустюмова И.П.,Рустюмова С.Т. Тренажер по математике для

подготовки к единому национальному тестированию (ЕНТ) Алматы:

Гарнитура «Таймс», 2011.-628б.

31 Фихтенгольц Г.М. Курс дифференциального и интегрального

исчисления. Т.1.Физматлит. – М. - 2003 г. – 680 с.

32 Демидович Б.П. Сборник задач и упражнений по математическому

анализу. М.: Наука. – 2000 г. – 624 с.

33 Зорич В.А. Математический анализ. М.: МЦНМО.. – 2019 г. – 576с.

76


ПРИЛОЖЕНИЕ А

Найти пределы:

2

1. x x2

lim ;

x2 x2

3x2

3.

5.

7.

3x42x3x25x5

lim ;

x1 x31

2 x

lim ;

x4 3 2x

1

2x2

x5

lim ;

x

3x2

10x1

9. x a x

11.

13.

lim ;

x

lim

2x2

3x

5

x1

lim x2

1

.

x0

2x2

x 1

15. lim 3x2

5x

2

.

x2

5x2

12x

4

17.

19.

21.

x2

2x

3

lim

.

x

2x2

3x

4

x3

5

lim .

x

x 2 3

2x

3

lim .

x

x 2 2

2 x

x21

x 3

23. lim .

25.

x 1

lim .

x1

1

x

2

x0

3

x 3x

x 2

27. lim .

29. lim

.

x3

x 3

2x

10

4

3

2. x 1000

lim ;

x10 x320x2100x

4.

x 63

lim ;

x3 x2

9

6.

8.

3

1x2

1

lim ;

x0

x2

10.

12.

14.

16.

18.

20.

22.

24.

3x4

2

lim ;

x

x3 3 x

4

3 2

lim x x

x

2

3x

;

4 3x

2

lim

3x

x2

x5

lim x2

3x

2

.

x1

x2

4x

3

lim

x1

2

2x2

5x

3

.

x2

6x

7

x 3

lim

x

2x2

3x

5x3

7x

lim .

x

1

2x3

2x3

4

lim .

x

x 2 5

lim

x0

a x

x

4 x 2

x16

x 4

26. lim .

28.

.

4

a

.

x2

lim

.

x0

x 2 4 2

30. lim x 3 2

.

x1

x 1

77


31.

33.

35.

37.

39.

41.

3 x a 3 a

lim

.

x0

x

lim

x

x2

4 x .

lim 1 2

x x.

x

cos( x

/ 2)

lim .

x1

1

x

cos6x

cos 4x

lim

.

x0

x2

lim tgx

tg2x

x

/4

.

sin

2

x tg

4

x

43. lim

.

x0

3x

2

5x

4

45.

47.

49.

x x

x lim sin 3 sin12

0

10x

x

lim 0

5 sin x

x

tgx x

x lim sin

0

sin3

x

51. (1 sin x) cos x

x

lim

0

53.

55.

x

5x3

2

lim .

5x3

x1

2x

3

lim .

x

2x

1

1

57. 2

lim (cos x)

x .

x0

x

59.

x

lim ln(1 4 )

0

x

sin( x 2)

71. lim .

x2

4x

8

5 sin x

78

32.

34.

36.

38.

40.

42.

44.

46.

. 50.

5x

lim

.

x0

3 1 x 3 1 x

2

lim x 4 x.

x

lim

x

sin 5x

lim

x 0 6x

2

1

x x.

1

cos x

lim

2

x .

x0

cos 2x

ctg2x

lim tg(

x)

.

x 0

4

x x

x

lim sin 4

0

x

sin 5x

sin 7x

lim

x0

x / 4

1 cos x 2

48. lim

.

x0

x

2

52.

lim

x0

x

.

1

cos x

3x

2x

lim .

x

2x

3

1

54. 2 2 3

lim (1 2x

) x

x0

56.

x2

x2

2

lim

.

x

x2

1

58. tg2x

lim (sin x)

.

x

2

. 60. lim xln(

x 1) ln x

x

72 .

arctg(2x

1)

lim

.

x1/2

4x2

1

.


73.

75.

77.

79.

81.

83.

85.

87.

89.

91.

93.

95.

97.

99.

arcsin( x 2)

lim

.

x2

x2

2x

ln x 1

lim .

xe

x e

ln( tgx)

lim .

x

/41

ctgx

lim xtgx.

x

/2

2

tgx

lim (sin x)

.

x

/2

1

cos x

lim

2

x .

x0

cos 2x

lim tgx tg2x

.

x

/4

5

32x5

x8

lim

x0

e5x

1

ln( x 1)

sin 2 x

lim

x0

arcsin x

2

x

lim

x5 x

sin 3x

lim

x 0

sin 5x

x

lim

2

x3

25

5

.

79

84.

86.

88.

. 90.

. 92.

. 94.

5x

6

x 3

2

x 2x

3

lim

x

2

2x

3x

4

lim

9 x

3x

2

x3

2

3

. 96.

. 98.

74.

76.

78.

80.

82.

. 100.

x 1

101. lim .

x

x 1

102.

103.

2

lim

x 4x

x .

104.

x

1 2

105. lim . 106.

x1 x 1

x 2 1

lntgx

lim .

x0

cos2x

lncosx

lim .

x0

lncos x

cos( x

/ 2)

lim .

x1

1

x

lim

x2

1

x2

1

( x 2) .

1

2

lim (cos x)

x .

x0

1

sin x xa

lim .

xa

sin a

ctg2x

lim tg(

x)

.

x 0 4

2tgx

x

4

x

2

lim

.

x0

arctg3x

4

lim

sin 2 xtg

x

.

x0

3x25x4

sin 7x

lim

x 0

3x

x0

2

2

.

sin 5x

lim

x

lim

x

2

.

x 2

x1 3

x

1

3x

4

lim

x

2

2x

3x

5

.

.

sin x cos x

cos x

lim .

x 2

4

lim

x

0

lim

x

1

x

x

1

2

x

1

x

x

3 2

x x

lim

2

3x

4 3x

x

2

.

.

.


107.

109.

111.

113.

115.

117.

119.

tg2x

lim . 108.

sin x

x

4

2

arctg(2x

1)

lim

1 2

x

4x

1

2

lim

x

x

3

3

1

х 4

lim

х

x

х 1

х

х1

. 100.

. 112.

. 114.

х

3

lim . 116.

3

x 1

lim

x1 sin( x 1)

ln cos x

lim

x

4 2

1

x

0

. 118.

1

. 120.

1

cos2x

lim

x

0 x sin x

arcsin( x 2)

lim

x

x2 2

x 2

2

х 1

lim

x

2

х

4

lim

x

x

4

.

2

х 1

lim .

lim

х

1

x x

x

x0

3

log

lim

x

x

3

3

x 1

3

.

2

.

.

.

1

.

80

Hooray! Your file is uploaded and ready to be published.

Saved successfully!

Ooh no, something went wrong!